Этого треда уже нет.
Это копия, сохраненная 6 ноября 2019 года.

Скачать тред: только с превью, с превью и прикрепленными файлами.
Второй вариант может долго скачиваться. Файлы будут только в живых или недавно утонувших тредах. Подробнее

Если вам полезен архив М.Двача, пожертвуйте на оплату сервера.
Тред тупых вопросов №110 SHAPLEY EDITION 497985 В конец треда | Веб
Тред вопросов о жизни, Вселенной и всём таком.

Спрашиваем то, за что в других местах выдают путёвку в биореактор. Здесь анонимные учёные мирового уровня критически рассмотрят любые гениальные идеи и нарисованные в Paint схемы.

Предыдущий тут: >>494296 (OP)
https://2ch.hk/spc/res/494296.html (М)

Q: Можно быстрее?
A: Можно упасть в пузырь альбукерке, наса уже почти надула его.

Q: Я начитался охуительных историй про уфологию, че делать, нам жопа?
A: Да, тебе жопа, можешь сгонять в зогач или куда оттуда пошлют.

Q: Что будет с человеком в вакууме без скафандра / если он упадет на черную дыру / попробует ступить на поверхность газового гиганта/солнца?
A: Он умрёт.

Q: Почему бы не привязать ракету к воздушному шару или стартовать с горы?
A: Космос - это не как высоко, а как быстро, большая часть энергии ракеты уходит на разгон вбок.
Подробнее тут https://what-if.xkcd.com/58/ (английский) https://chtoes.li/orbital-speed/ (перевод)
2 497991
>>497985 (OP)
Ах ты! Я хотел сделать перекат, а ты опередил на пару минут.
3 497993
>>497967

>но материя то одна у них


Почитай про волны, штоле.
4 497995
>>497985 (OP)
Ебаный стыд.
Во первых абдулькерка.
Во вторых не надуть а надуть.
В третьих интерференция.
5 497996
>>497995
Бред какой-то
6 497998
>>497985 (OP)
Ебанный стыд...
Во-первых, Алькубьерре.
Во-вторых, не упасть, а создавать вокруг корабля изнутри (иначе кина не будет).
В-третьих, НАСА искривляет пространство на десятимиллионную часть, контролируя это сверхточными интерферометрами, до самого варп-привода здесь - как до Антарктиды раком.
7 498001
>>497996
Не обращай внимания, у тру-вахтера в таком треде развлечений не так много одни тупые ответы, да тупые ответы, так что пасты после переката - это важный ритуал освящения треджа.
8 498002
>>497991
О, есть желающие перекатить? Я только за, следующий раз твой тогда.
9 498004
>>497866
>>497861
Система координат на картинке привязана к оси и экватору Земли, а не Солнца, наклонение эклиптики к солнечному экватору не имеет вообще никакого значения.
image.png1,4 Мб, 1100x812
Планета в режиме радиомолчания 10 498015
Космоанон, а вот была (есть?) такая идея, что мы сможем засекать инопланетные цивилизации по радиосигналам с их планет. Дескать, развились - изобрели радиосвязь - стали светить радиоволнами во все направления - мы их увидим и поймём, что там кто-то есть.
А что если по той или иной причине необходимо, наоборот, спрятаться от такого обнаружения?
Собственно: если человечество сегодняшнего образца не будет использовать радиосвязь во всех её видах, отключит радиотелескопы, вай-фай, блютусы и т.п. - но при этом продолжит пользоваться остальными технологиями (насколько это в поставленных условиях возможно): связь по проводам, электростанции работают, ЛЭП стоят, корабли ходят, промышленность промышляет; то насколько наш радиоволновой "фон" будет подозрительным? Неотличимым от естественного или мы и без радио нафоним достаточно, чтобы нас заметили?
Потенциальных инопланетян, чтобы не скатывать их во всемогущество, примем примерно равными нам самим, только сидящим где-нибудь в сотне световых лет отсюда.
11 498017
>>498015

> Собственно: если человечество сегодняшнего образца не будет использовать радиосвязь во всех её видах, отключит радиотелескопы, вай-фай, блютусы и т.п. - но при этом продолжит пользоваться остальными технологиями (насколько это в поставленных условиях возможно): связь по проводам, электростанции работают, ЛЭП стоят, корабли ходят, промышленность промышляет; то насколько наш радиоволновой "фон" будет подозрительным? Неотличимым от естественного или мы и без радио нафоним достаточно, чтобы нас заметили?


Радиотелескопы это вообще приемники, а не передатчики, а вайфаи с блютусами хуй покинут атмосферу с их потешными мощностями, большая часть сигнала на больших расстояниях будет неотличима от фонового шума

> Потенциальных инопланетян, чтобы не скатывать их во всемогущество, примем примерно равными нам самим, только сидящим где-нибудь в сотне световых лет отсюда.


Они нихера и не увидят, мы активно используем радио только с половины 20 века
12 498020
>>498015
эта концепция это бред
по факту мощности излучения такие слабые, что обнаружить их можно только если смотреть точно на планету, причем с расстоянием сигнал рассеивается квадратично
13 498023
>>498020
Эх, сейчас бы мнение мелкобуквы послушать!
Был основан некоммерческий Институт SETI и запущен проект «Феникс»; проект предусматривает изучение тысячи ближайших звезд солнечного класса в радиодиапазоне 1200—3000 МГц. В этом проекте используются чрезвычайно чувствительные приборы, способные уловить излучение обычного аэродромного радиолокатора с расстояния в 200 световых лет.
14 498024
>>498023
Сети давно скатились в лженауку и осваивание денег, это бред.
Для обнаружения радара за 200 св лет нужен направленный приемник с аппертурой в километры, возможно даже десятки. Причем фокус с синтетической аппертурой тут не прокатит.
15 498026
>>498024
Ты это можешь с циферками посчитать, скажем для 10 КВт мощности радиолокатора и расстояния 200 св лет? Частоты вон постом выше.
16 498027
>>498023

> Был основан некоммерческий Институт SETI и запущен проект «Феникс»; проект предусматривает изучение тысячи ближайших звезд солнечного класса в радиодиапазоне 1200—3000 МГц. В этом проекте используются чрезвычайно чувствительные приборы, способные уловить излучение обычного аэродромного радиолокатора с расстояния в 200 световых лет.


Микроволны разве не поглощаются атмосферой хорошо ? Я не думаю что кто-то намеренно будет жарить мега/гигаватты в воздух
17 498028
>>498017

>Они нихера и не увидят, мы активно используем радио только с половины 20 века


Ну, блин, очевидная же условность.
Предположим, что мы в таком радиомолчании сидим уже давно.

>большая часть сигнала на больших расстояниях будет неотличима от фонового шума


С блютусом то шутейка была; но, допустим, те же ЛЭП? Собственно, главное, что вызывало у меня опасения - это ЛЭП и электростанции.

>такие слабые, что обнаружить их можно только если смотреть точно на планету


Возможно, они что-то подозревают?
18 498030
>>498028

> С блютусом то шутейка была; но, допустим, те же ЛЭП? Собственно, главное, что вызывало у меня опасения - это ЛЭП и электростанции.


50 Гц хуй поймаешь (нужны гигантские антенны просто) и в лэп там вообще звуковой диапазон, слышал же как гудят они
15241104027760.jpg29 Кб, 460x405
19 498037
>>498030
В целом, кажется, понятно.
Спасибо, анон.
20 498046
>>498026
мне впадлу сейчас
видел на ютубе нужное, ищи radio silence или что-то такое, там цифры были
21 498056
Есть у меня тупой вопрос который мне покоя не дает. Вопрос этот про верх и низ. Моя проблема в том что мне кажется что мы запускаем ракетонасители и летаем в одном направлении, то есть я хочу сказать в какой стороне все относительно земли? Есть какая нибудь инфографика? Там луна выше например, солнце выше, марс внизу, юпитер ниже это пример никаких утверждений. Моя идея в том что у космоса возможно есть дно. Прямо такое себе дно осязаемое. Буду рад вашим аргументам.
22 498057
>>498056
Никакого дна нет пространство бесконечно расширяется во всех направлениях
23 498058
>>497985 (OP)
Что на второй пикче?
24 498064
>>498058
Читать умеешь, хотя бы со словарем?

Это шаровые звездные скопления Млечного пути, а картинка показывает их распределение. По ним Харлоу Шепли и установил впервые как размеры нашей галактики, так и место Солнца в ней.
25 498065
>>498056

> в какой стороне все относительно земли?


Большинство интересных тут людям вещей (навроде Марса) - снаружи.
Хотя некоторые вещи (например, ядро Земли) - внутри.

Верх и низ в простом житейском понимании этих слов существуют тогда, когда тебя куда-нибудь тянет. Вот куда тянет - там низ. А с противоположной стороны - верх.
Когда стоишь в Испании - низ в той стороне, где Новая Зеландия. То есть, скорее всего, у тебя под ногами - если стоишь по-обычному, а не на голове или ещё как.
Когда ты в Новой Зеландии - низ, соответственно, в стороне Испании.
А когда падаешь на Юпитер - низ в стороне Юпитера, совершенно независимо от того, где в это время в результате сложного танца объектов Солнечной системы оказались Испания и Новая Зеландия.

Если же у тебя другое, не житейское определение низа - то сначала сформулируй это определение: "низом я буду называть то направление, которое..." - а потом будем дальше разбираться.
26 498067
>>498015

>А что если по той или иной причине необходимо, наоборот, спрятаться от такого обнаружения?


>


https://habr.com/ru/post/385543/
27 498097
>>497995
Инфляция пасты
28 498109
>>498097
Наоборот, коллапс с элементами raspeedorusheevania.
Solntse-i-Betelgejze.png472 Кб, 914x597
29 498177
До Бетельгейзе всего 200 парсек и она вот-вот взорвется как сверхновая типа II. Достанет ли до нас смертельный поток излучения и плазмы?
30 498178
>>498015

>инопланетные цивилизации


Они уже спалили нас.
31 498194
>>498178
иди нахуй
>>498177
ну смотря как понимать достанет. ночью светло наверное будет, но ничего опасного
32 498196
>>498177

> и она вот-вот


Ну как вот-вот, может прям сейчас, а может и через пару тысяч лет ебануть.
1280px-Eso1737e.jpg215 Кб, 1280x865
33 498200
>>498194

>иди нахуй

34 498213
>>498177
Бетельгейзе всего где-то в 15 раз больше Солнца по массе, даже если по картинке кажется, что в мульон. Ее просто распучило на старости лет. Солнце тоже будет пучить (плюс-минус до орбиты Земли), хотя и не так сильно.
Pan-Flyby.gif3 Мб, 1024x1024
35 498221
>>498200
иди нахуй
36 498286
>>498177
гамма-всплеск может привести к вымиранию - https://ru.wikipedia.org/wiki/Ордовикско-силурийское_вымирание
37 498330
>>498213

>Бетельгейзе всего где-то в 15 раз больше Солнца по массе, даже если по картинке кажется, что в мульон. Ее просто распучило на старости лет.


Спасибо, Кэп.
38 498345
>>498221
Пельменей захотелось.
Atlas2017-04-12rawpreview.jpg75 Кб, 648x518
39 498375
>>498345
Там же еще есть
40 498394
>>498375
У меня встал.
41 498488
>>498394
Выебал бы?
inb4 ты бы и S/2009 S 1 выебал
42 498605
Съемка падения метеорита https://youtu.be/SLT0pFiSLbE
43 498615
>>498605
У единственного чувака оказалась камера с зумом (на 0:45) https://youtu.be/_UKsMp9pj8o
44 498616
>>498605
>>498615
Поздравляю с разморозкой няш, но мы это ещё 5 лет назад посмотрели.
45 498619
>>498616
Я люблю вспоминать периодически. Реакции и универсальное "нихуя себе" доставляют
fullsize.jpg38 Кб, 600x420
46 498650
Почему Буран с воздушным стартом с Ан-225 не может заменить обычные ракетные пуски? Ему даже космодром не нужен.
47 498652
>>498650
Потому что чтобы выйти в космос Бурану нужен охуиллион тонн топлива и йоба-двигатели, которые могут ебать гравитацию и воздушное сопротивление в рот. А на Ан-255 есть место только для Бурана.
maxflig.gif26 Кб, 439x149
48 498661
>>498652
Но ведь МАКС же разрабатывали:
http://www.buran.ru/htm/maxmain.htm
49 498664
>>498650
Потому что воздушный старт Бурана с Ан-225 - это максимум планирование Бурана на землю.
Всякий проект воздушного старта упирается в то, что стартовая масса РКН ограничена грузоподъёмность самолёта.
>>498661
МАКС очень интересный, да.
50 498668
>>498650

> Почему Буран с воздушным стартом с Ан-225 не может заменить обычные ракетные пуски? Ему даже космодром не нужен.


Потому что Ан-225 добавляет в лучшем случае жалкие 250м/с дельты, которые и ссаной пороховой петардой добавить можно.

Чтобы от воздушного старта был толк, носитель должен быть гиперзвуковым. Ну или хотя бы чем-то уровня Миг-31. Но никак не слоупочной дозвуковой коровой. Единственное, что корова может сделать полезного - это долететь до экватора и там пульнуть, но это слишком маленькое преимущество, чтобы городить ради него такую хуйню.
51 498682
>>498661
Братан, если ты не заметил, МАКС сильно меньше, и , что логично, легче, чем огромный ёбаный Буран.
52 498688
Я вот не могу понять. Почему взорвалась сингулярность? Случился момент взрыва, но ведь для наступления момента нужно время, а его ведь не было.
53 498743
>>498688
Да никто не знает, епта. Только теории. Если с хокинговской точки зрения (как я ее понимаю, не бейте), то в любой момент времени сингулярности не было, сразу с первой же секунды была нормальная расширяющаяся вселенная. Правда при t -> +0 (ага, при уменьшающемся времени) она "стремилась" к сингулярности, но ее не достигала.
54 498744
>>498688
Не стоит пытаться понять. Ты же не пытаешься понять как боженька за пять дней сделал вселенную, ты ведь не забиваешь себе голову такими маняфантазиями. Так и тут.
55 498750
>>498743
Раз мочоные наконец дофантазировались до этого, то я вам одну умную вещь скажу, только вы не обижайтесь.

Вот это вот:

>с первой же секунды была нормальная расширяющаяся вселенная. Правда при t -> +0 она "стремилась" к сингулярности, но ее не достигала.


ничем не отличается от стационарной вселенной без расширения но с усталыми фотонами. Вселенная типа всегда нормальная, фотоны "краснеют", сингулярности не было никогда. То есть расширение кажущееся, на самом деле его нет.
sage 56 498756
>>498750
А вот и ебанавт с хронической усталостью металла пожаловал.
57 498765
>>498756

И долбоеб с сажей тут как тут
58 498793
Как спц относится к Циолковскому? Основоположник и теоретик космонавтики, вдохновивший Королева или поехавший дед, решивший решенный сто лет до него диффур и придумавший очевидные вещи типа многоступенчатости и оранжерей на космических кораблях, не проработав их детально?
sage 59 498801
>>498793
спейсач не один человек, тут у каждого своё мнение, так что сама постановка вопроса не верная.
60 498809
>>498793

>спц


Сербская православная церковь?
61 498812
>>498809
Союз порнографов-целителей
62 498843
>>498793
Циолковский - прообраз обитателей /spc/, в одиночку нагенеривший контента на несколько омских КБ и омской физики тредов. А ещё Эдуардович дико котировал трансгуманизм и евгенику. Короче, на бордах был бы уважаемым человеком.
63 498869
>>498843
На бордах уважаемых людей нет, пидор
64 498923
>>497985 (OP)
Почему ровные пацаны ставят на двигатели бустерные насосы, а хряптор без них?
65 498977
Аноны, вопрос про Пульсары, там же притяжение пиздец, а оболочка у него жидкая или твердая? Они же пиздец какие небольшие но охуеть тяжелые, что там за вещество? Если я пролетая мимо отколупаю себе кусочек в качестве сувенира, то скока будет весить кусок размером с мяч теннисный?
66 498986
>>498977

> оболочка у него жидкая или твердая?


Мир не ограничивается жидкостями и твердыми веществами. Обычные звезды состоят из плазмы, разновидности газа. Нейтронные звезды состоят из вырожденного газа, если я ничего не путаю.

> Если я пролетая мимо отколупаю себе кусочек в качестве сувенира, то скока будет весить кусок размером с мяч теннисный?


Дохуя, наверно. Посчитай сам. Думаешь, нам не лень считать за тебя?
Neutronstarcrosssection.svg.png279 Кб, 1920x1105
67 498993
>>498977

> то скока будет весить кусок размером с мяч теннисный


Загугли какой нибудь пульсар посчитай плотность и умножь на объём своего мячика получается крайне дохуя ( у меня получилось ~8 масс, а точнее ~7.6 масс Земли если взять PSR B1919+21

> а оболочка у него жидкая или твердая


пикрил
68 498999
>>498993
Алсо, без гравитации пульсара (то есть, если отколупанное попробовать увезти как сувенир), это пизданет на всю ивановскую и нихуя уже не будет иметь размер теннисного мяча.
69 499000
>>498999
Я так сейчас и представил как наши туристы в космосе долбают пульсар, и с криком "ебанный врот" он взрывается оставляя гамма всплески, а в новостях, "сегодня на неопределенный срок закрыта космотуристическая зона 3XMM J004301.4+413017"
70 499008
>>498993

>у меня получилось ~8 масс, а точнее ~7.6 масс Земли


Проверять я, конечно же, не буду. Но интуиция подсказывает, что ты на несколько разрядов наебался.
71 499009
>>499008
Похуй, по бырику прикинул пульсар размером в 10 км диаметром и массой в два соулнца, и мячик диаметром 10 см для простоты расчета. Итого кратность пять разрядов, для объема третья степень. Получается в пульсаре поместится 1015 таких мячиков. Сколько там масс Земли в Солнце? 2х1030/6х1024. В общем не знаю как ты так считал.
72 499024
>>499000

>Не лезь, дебил, она тебя сожрет!


>Бляаааааадь!

73 499041
>>498993
Спасибо больше анончик
74 499053
>>498986

>Дохуя, наверно. Посчитай сам.


Я тупой гуманитарий, поэтому и сюда пришел.
75 499151
Кто слышал о теории, что наша вселенная - ящик? Вроде поляк придумал.
76 499152
>>499151
Слышал о теориях, что наша Вселенная поддон, торшер, треугольник, больничная утка, микроволновка, бутылка Клейна, тапок, ромбокубоокатаэдр, анус Галкина, спасательный круг с борта эсминца USS Wickes, CTR00AO7N2545dghe97jhy-tmc, свёрнутый ковёр, плавленный сырок. Но такую хуйню, чтобы ящик - впервые.
77 499154
>>499152
Сам в недоумении. Что-то он там увидел же.
78 499156
>>499152

>больничная утка


Это многое бы объяснило, на самом деле. У этой теории даже предсказательная сила есть.
79 499157
>>499154
Ну, в ящике всякое можно увидеть, особенно если попробовать таблетки, которые дед в нём хранит с 47-го года.
Raptor-test-9-25-2016.jpg1,5 Мб, 4752x3168
80 499234
>>498923
Бамп
81 499236
>>497985 (OP)

>Milky way.mp4


Почему на этом говне угловая скорость везде одинакова?
82 499237
>>499236
Потому что это не галактика вращается, а наблюдатель летит вокруг неё.
ферштейн?
83 499244
>>499234
Что такое "бустерный насос"? Турбонасосы для топлива везде стоят.
84 499253
>>499244
Вангую, что этот анон перепутал раптор с супердракой. А на супердрако вытеснительная подача, чтобы мгновенно выходить на максимальную мощность.
85 499254
>>499236
потому что это из игры какой-то заставка. А там на нее пох всем - она скрыта за буквами меню и видишь ты там ее секунд 10
86 499285
>>498793
В детстве читал книжку с его фантазиями и заинтересовался космосом. Считаю Циолковского хорошим человеком.
87 499316
>>499253
Тогда ещё непонятнее, потому что никакие "ровные пацаны" на подобные двигатели насосы не ставят.
88 499339
>>499254
Не знал, что телескоп Gaia это игра какая-то. Видео в оп-посте это визуализация с его данных.

https://www.spacetelescope.org/videos/heic1905b/
89 499362
Вот смотрите через 5 миллиардов лет солнце распухнет и поглотит землю. Земля само собой расплавится и упадёт к центру звезды.
Но вот если брать звёзды раздувающиеся куда сильнее. И вот планета Н(a) поглощается звездой Н, но внешний слой звезды так разряжен, что планета не сможет затормозить свою орбитальную скорость за оставшееся звезде время и после сброса оболочки масса расплавленного нечто будет плавать на орбите вокруг того что осталось. Может ли быть такое?
90 499364
Если выковырять внешние слои нейтронной звезды, можно ли будет сохранить её хоть при каком-то меньшем давлении или она в любых условиях будет расширятся?
91 499365
>>499364
Нейтроны там стабильны только из-за чудовищной гравитации, в свободной форме они через 10 минут распадаются
92 499366
>>499365
Ни я говорю о том слое, где ещё железо.
93 499370
>>499362

>внешний слой звезды так разряжен, что планета не сможет затормозить свою орбитальную скорость за оставшееся звезде время


Я тут из разряда тупых, а не экспертов, но разве при такой разряжённости там смогут идти термоядерные процессы?
94 499371
>>498923
Кавитация
95 499373
>>499371
Кавитация что?
96 499378
>>499370
Но термоядерные процессы идут ближе к ядру. А у границы там каких-то 3000К.
97 499382
>>499364
Ее - кого? Кусочек коры ты не оторвешь, а если оторвешь магически, то не сохранишь, пока не скомпенсируешь огромное, на десяток с лишним порядков падение силы гравитации, которая держала вместе огромную плотность. Сама звезда отколупывпния кусочка не.заметит, оче быстро вернется к сферической форме.
98 499385
>>499364
Нет. Причем не сохранишь даже в сантиметре от поверхности звезды: по вертикали-то гравитации будет достаточно, разница небольшая, а вот по горизонтали исчезнет давление соседних "кусков" звезды, и выковырянный кубик "растечется" во все стороны.
99 499392
>>499366
Железа тоненький слой всего, он неважен на самом деле.

В любом случае, настоящий ответ — можно. Если взять уже остывшую звезду и скрести очень аккуратно, то примерно до 0,2-0,1 солнечных масс можно добраться (естественно, таких звезд не бывает в реальности, потому что нет процессов, по которым они могли бы образоваться).

Как только этот нижний предел будет пройден, то да, тут же начнется распад нейтронов обратно в протоны и электроны и катастрофическое разжатие остатка.

http://adsabs.harvard.edu/abs/1993ApJ...414..717C
http://adsabs.harvard.edu/abs/2006IJMPA..21.1555B
100 499415
Вселенная расширяется? Это известно на 100%?
101 499441
>>499415
Да, если, конечно, не выснится, что фотон устает, или подобная дичь
102 499443
>>499441
А если она расширялась в прошлом, ведь чем дальше от Земли тем быстрее расширение, но чем дальше тем и более "прошлую" вселенную мы наблюдаем, а на сегодняшний момент, на самом деле, расширение закончилось?
103 499444
>>499441
А расширяется она внутрь себя?
104 499445
>>499362
Ну да, вполне может. При соблюдении десятка редчайших условий, но может, хотя орбита безусловно изменится. Просто для понимания редкости условий я тебе перескажу теоретическую модель. Проходя по пути эволюции звезда, запуская новую стадию синтеза, углеродного, например, увеличивает светимость. Что в случае с планетой, обращающейся по орбите с малым эксцентриситетом, образованной из протопланетного облака, как наша Земля, приводит к первым траханьям этой самой орбиты. В первую очередь из-за увеличения светимости - следовательно увеличения светового потока от звезды. В контексте планетарной массы это копейки, но ведь это и воздействия на миллионы лет. Во вторую очередь нестабильность звезды. Пертурбации вещества в объеме звезды приведут к появлению заметных гравитационных аномалий, аналогичных масконам, что только еще активней распидорасит орбиту планеты. Эти же пертурбации обеспечат постоянные колебания степени воздействия светового потока.
Ну и еще должно так совпасть, чтобы планета не попала в плотные слои звезды, которые в случае со сверхгигантами пусть и экстремально разряжены, но которые за миллионы лет точно остановят бег любого орбитального гуляки. Это должно быть попадание в верхние слои фотосферы, в которых планета будет периодически ласкаться гигантскими протуберанцами, но не будет двигаться прям в теле звезды круг за кругом. Ах да, еще нужно учитывать, что планета сначала проебет всякую атмосферу, а затем проебет кучу вещества, даруя его как протуберанцам, так и звездному ветру. И эти изменения планетарной массы тоже внесут вклад в расшатывание орбиты.
Кароче это из разряда тех же вероятностей, что например захваченная звездой блуждающая планета вдруг под воздействием столкновений с телами системы приобретет околоидеально круговую орбиту. Или из разряда вероятностей, что ты выиграешь в лотерею миллиард и перестанешь задаваться подобными вопросами, а жизнь пойдешь жить. Теоретически-то возможно...
105 499447
>>499445

>разряжены


Бля, ну позорище, пздц. Впрочем, кто понял, тот понял.
106 499471
>>499445
У меня для Земли во внешних слоях звезды плотностью 1/1000 воздуха при н. у. получается годовое ускорение порядка 100 м/с^2 (тупо воткнул в формулу лобового сопротивления). Сдается мне, что долго она так не полетает. Или там плотности еще на порядки ниже?
107 499475
>>499471
Хотя да, с плотностью переборщил. Это на порядок больше даже средней плотности Солнца, раздувшегося до орбиты Земли, а ведь у поверхности плотность еще гораздо меньше средней.
108 499479
>>497985 (OP)

>пузырь альбукерке



Это какой-то локальный мемчик или просто оп-хуй из переката в прекат копирует очепятку?
109 499481
>>499479
Пузырь Аль-Букакке
110 499485
>>499444
В каждой точке во всех направлениях в каждый момент времени. Мне тоже некоторые моменты непонятны. Расширяется же только пространство, а не вещество в нем? Растёт ли расстояние между галактиками? Между Землей и Луной? Между мной и тобой? И если растёт, то насколько?
111 499500
Почему НАСА, научная организация с крупнейшим бюджетом в мире, не осиляет разработку автоматической стыковки и использует КанадАрм, в то время как Совок осилил тестовую автоматическую стыковку еще в середине 60х и активно начал применять в 71 на Салюте?
Блять, ну если не допираете, как сделать, слишком трудно это для вас - спиздите просто систему с Союза. 3 радиоисточника/приемника - и готово. Вы че, ребят? Вы что, живете на плоскости блять?
Позор же. Уже 50 лет СССР может стыковаться автоматически, без манипуляторов - эти чуханы все еще не осилили. С крупнейшим-то в мире бюджетом. Вот это попил бабла.
112 499518
>>499500
Ты сперва ответь нахуй вообще необходимость в автоматической стыковке
113 499519
>>499500

>50 лет


>СССР


Он утонул 28 лет назад, как он может все еще автоматически стыковаться?
114 499525
>>499500
также как и посадка спейс-шаттла в ручном режиме была - есть вся автоматика, всё блядь отлажено на 100 раз, но ПИЛОТАМ ЧТОТО НАДО ДЕЛАТЬ, как-то, же, блядь, их значимость показать. Вот в ручном и проводят. Автоматическая система есть, просто людям в падлу просто так летать, поэтому и договоренность имеется такая.
115 499530
>>499525
Дурацкая затея на самом деле, один раз уже охуенно так состыковались вручную — аж станцию протаранили, продырявили корпус и навсегда угробили целый отсек. Правда, тоже совки, а не американцы.

https://ru.wikipedia.org/wiki/Прогресс_М-34
116 499535
>>499518
Ну, это в дохуя раз удобнее, надежнее и безопаснее, нет?
117 499539
>>499535

>удобнее


нууу, это не самое важное в космонавтике

>надежнее и безопаснее


чем? при захвате рукой корабль не делает нихуя
а при активной стыковке есть много работы двигателями, что-то заест или не сработает - корабль протаранит станцию
118 499562
>>499479
Охуел? Хайзенберга не уважаешь?
119 499571
>>499530
Русня, хули.
120 499572
>>499371
Знаю, что кавитация. Почему нормальные пацаны с ней борются, а Муск - нет?
image.png932 Кб, 1023x801
121 499573
>>499572
Кавитация - это маленькая смерть, влекущая за собой полное уничтожение.
Я встречусь лицом к лицу со своей кавитацией.
Я позволю ей пройти через меня и сквозь меня.
И, когда она уйдет, я обращу свой внутренний взор на её путь.
Там, где была кавитация, не будет ничего.
Останусь лишь я.
image.png724 Кб, 898x634
122 499574
>>499253
Учи матчасть чтобы не быть батхертом. БНАО и БНАГ стоят на РД-170 например, бустерные насосы не тождественны ТНА а являются его частью.
>>499373

>В то же время давление подачи и частота вращения ротора ТНА схем с «дожиганием» значительно выше. Значительный рост всех параметров ТНА наблюдается с первого мощного двигателя РД253. Это обусловлено повышением давления в камерах сгорания двигателей (с 80-90 кгс/см2 для «открытой» схемы до 150 кгс/см2 в двигателе РД253 и до 250-270 кгс/см2 в двигателях РД170 и РД180). Вызванная этим необходимость значительного повышения напора насосов, в свою очередь, потребовала соответствующего повышения частоты вращения для снижения массы и уменьшения габаритных размеров ТНА. Это оказалось возможным благодаря применению автономных бустерных турбонасосных агрегатов, обеспечивающих необходимое повышение давления на входе в основные насосы.


Как я понял из статьи энергомаша и односложного ответа >>499371 , бустерные насосы нужны чтобы основной насос, вращающийся с ебической скоростью и создающий ебическое давление не начал кавитировать от того, что в него из бака самотеком пришла жидкость слишком низкого давления. А может быть дело в том, что бустерные насосы шнековые, а основные - центробежные, хрен его знает, я гидродинамику не изучал.
Пикрл - БНАО РД-170.
http://www.lpre.de/resources/articles/Energomash2.pdf
sailorvenusbyamishanda.jpg81 Кб, 600x821
123 499605
Что почитать школьнику о Венере? Статьи в Википедии и на Лурке уже прочитал, хочу ещё!
124 499606
>>499362
А можно Землю как-то подвинуть, чтобы сохранить культурный слой?
125 499608
>>499500
Как и на железной дороге, наша автосцепка лучше.
126 499620
>>499606
Тебе нельзя. А в принципе запрета на это нет, есть масса способов, каждый из которых требует фантастических масштабов энергий.
127 499630
>>499605
Энциклопедия для детей по астрономии от издательства "Аванта+", глава про Венеру. На RuTracker где-то лежала, поищи самое свежее издание.
Несмотря на название "для детей", там очень хорошие учёные грамотно писали - просто, но интересно, с самыми современными данными.
128 499642
почему нельзя построить станцию,вывести ее на орбиту,а потом второй ракетой вынести доп двигатель,состыковать его и запустить?тогда по идее скорость станции будет намного больше,чем если бы ее запускали обычным способом с земли?
129 499661
Господа, что за мазафака написана в Википедии в статье про Сатурн: "Магнитное поле является почти дипольным, так же как и у Земли, с северным и южным магнитными полюсами. Северный магнитный полюс находится в северном полушарии, а южный — в южном, в отличие от Земли, где расположение географических полюсов противоположно расположению магнитных"? В смысле? То есть у нас Арктика лежит на южном магнитном полюсе? Тогда почему бы не переименовать северное географическое полушарие в южное, раз оно по сути является южным?
130 499663
>>499642
а нахуй тебе большая скорость? чтобы потом до нее лететь сложнее было?
131 499665
>>499661
Потому что географические полушария не меняются, а планетарное магнитное поле регулярно ездит туда-сюда.
132 499668
>>499663
что бы быстрее добраться до нужного объекта,очевидно же
133 499675
>>499668
Че? До какого объекта?
134 499692
>>499605
"Страна багровых туч"
135 499693
>>499675
до урана например
Координаты магнитных полюсов.png7 Кб, 1124x143
136 499694
>>499665
Понятно, но почему написано "расположение географических полюсов противоположно расположению магнитных", хотя, на самом деле они почти рядом?
137 499723
>>499694
Называется он "северный магнитный полюс", но это южный полюс(то биш плюс) ибо к нему притягивается северный полюс компаса(то биш минус). Всего лишь названия, не заморачивайся. Значение северного полюса, как южного магнитного нужно лишь геофизикам. Если ты географ, то там северный полюс и точка.
138 499822
>>499723
Понял, благодарю.
139 499940
Какой может быть максимальный размер ракеты?

Ведь при прочих разных, чем больше ракета, тем дешевле доставка на орбиту в пересчете на килограмм, так какого размера можно ее построить? 10к тонн? 100к? Еще больше? Или Сатурн V со своими 2970 тонн стартовой массы уже приблизился к конструкционному пределу?
140 499974
>>499940
Ващет собирались изначально делать в два раза больше чем Сатурн-5, чтоб сразу на Луну лететь, без орбитера с лэндером, потом уже поупырили мел.
141 499989
>>499940

>Какой может быть максимальный размер ракеты?


Какой угодно, хоть миллиард тонн.

Масштабирование в высоту упирается в гравитацию и максимально возможную башню, которую на данный момент можно построить. Сегодня это километр, хотя может быть и выше.
Масштабирование вбок не ограничено по большому счёту ничем кроме геометрической целесообразности.
142 499991
У кого система дальнекосмической радиосвязи в 20 веке была круче: пендосовский Deep Space Network или совковый Центр дальней космической связи?
143 500033
>>499991
У совков была плохая электроника для дальнего космоса. У РФ кстати её уже нет, потому что повторить эту совковую технологию не могут. Что бы долететь до Луны этой электроники хватало, до Марса уже нет.
144 500064
>>500033
Какую технологию, кремний на сапфире, с тренчами, ортогональностью и частичным перекрытием? Это в любом гараже делают.
Не повторяют за отсутствием целесообразности 22крад совка уже бессмысленно, 100крад позволят слетать разве что к марсу, мегарад не достижим без ноу-хау инфинеона.
145 500119
так я не понял,Юпитер это просто газовый шарик,без твердрй поверхности?
146 500120
>>499642
Формулу Циолковского не наебать в любом случае, ну если ты только световой парус воткнешь и будешь лазером разгонять.
147 500122
>>500119
Из твердого там только ядро.
148 500123
>>500122
оно горячее?
149 500124
>>500123
Естественно.
150 500125
>>500124
и какова его доля?
151 500126
>>500122
>>500123
а почему так получилось?
152 500127
>>500126
Распад радиоактивных элементов полагаю, у Земли, по крайней мере так.
153 500128
>>500125
Я откуда знаю? Раздели массу ядра на массу планеты.
154 500130
>>500127
я про о что он газообразная планета
155 500131
>>500130
Наверное, газов в протопланетном диске было намного больше, чем металлов и неметаллов.
156 500132
>>500131
Тут даже наоборот - во внутренней части солнечной системы доля газов меньше, поскольку их снесло солнечным ветром.
На водород и гелий приходится около 98% от массы всего вещества во вселенной.
157 500133
>>498015
Это как ебаться в гондоне.
158 500134
>>499642
Можно. МКС без регулярных попуков двигателей пристыковавшихся кораблей сходит с орбиты из-за диссипации.
159 500135
>>500134
корректировка это немного другое
160 500144
>>500126
Просто он достаточно тяжелый, чтобы притянуть водород и гелий. У Земли тоже была газовая оболочка, но ее сдуло.
161 500165
как так,свет шел от галактики 13.3 млдр лет,а расстояние до него 32 млдр св лет?обычно говорят про первую цифру при обсужд расстояния
162 500172
>>500165
Расширение пространства. Свет шел год, а с тех пор этот световой год растянулся в разы.
1382199236632.jpg381 Кб, 1818x1302
163 500226
А >>500000 гет проебали уже?
15600802096492.jpg54 Кб, 700x525
164 500229
Интересно, а почему никто не пытается создавать космические миссии с помощью краудфандинга? Я думаю что из семи миллиардов найдется достаточно людей чтобы задонатить на запуск ракеты допустим к Урану или Нептуну.

>Как и с Ураном, механизм нагрева неизвестен, но несоответствие большое: Уран излучает в 1,1 раза больше энергии, чем получает от Солнца. Нептун же излучает в 2,61 раза больше, чем получает, его внутренний источник тепла добавляет 161 % к энергии, получаемой от Солнца. Хотя Нептун — самая далёкая от Солнца планета, его внутренней энергии оказывается достаточно, чтобы породить самые быстрые ветры в Солнечной системе.

165 500230
>>500229

>а почему никто не пытается создавать космические миссии с помощью краудфандинга?


Про лохотронщиков Mars One и "самый яркий Маяк на небе" почитай, даун.
166 500233
>>500172
Как расширение может быть быстрее скорости света, если превышение скорости света - это нарушение закона причинности и не может существовать?
167 500239
>>500229
Че там ловить интересного у газовых и ледянных хуевин, интересны могут быть только их спутники
168 500240
>>500239
Да вот есть научный интерес, а какой интерес может быть кроме научного? Может первонах, если кто-то сделает что-то эпическое типа межпланетной миссии, то его имя войдет в историю и может его именем назовут какой-нибудь астероид или кратер на спутниках. Это после смерти, а еще при жизни может быть интерес в славе раздавать интервью и привлекать внимание, пиздеть что делаешь это потому что хочешь найти там жизнь, новые открытия.
Еще можно тестить технологии, с намеком что тестируя технологии ты потом их будешь развивать. Что именно можно тестить так далеко? Системы связи, телескопы, системы электричества, но к газ-гигантам долго лететь, пока результаты тестов придут, уже времени пройдет что уже что-то можно будет с нуля круче придумать? Можно конечно получить охуенно крутые фото, которые будут доказательством охуенности твоего товара. Если бы какая-нибудь японская фирма послала миссию к Ио и сняла бы планету в охуеннейшем разрешении, то они могли бы создать просто отпадную рекламу "До и После фотографий нашими камерами". Создать 3D модели и глобусы ИО по материалам своей камеры и продавать. Создать связь бренд+небесное тело, чтобы при воспоминании о небесном теле вспоминали бренд. Как Маск почти добился что его вспоминают когда упоминают Марс в умах быдла.

Можно делать вид что пытаешься открыть полезные ископаемые, имеет смысл лишь в сотрудничестве с кем-то кто их будет добывать или если сам будешь в последующем миссии отправлять по посадкам и добычам.
Еще есть мини пиар - посылка архивов, днк, праха, имен, лейблов, брендов, показ их.
Нужно думать над многими этими вариантами, но краудфандинг это конечно дно говна, гораздо лучше чтобы среднего уровня компания сама поставила себе такую цель. Дисней кстати мог бы в одно рыло потянуть космическую программу уровня Аполлон по ресурсам.
169 500241
>>500233
В каком смысле быстрее скорости света? Пространство никуда не двигается.

Нельзя по пространству перемещаться с большей скоростью, а с самим пространством что угодно может в это время происходить, никаких законов растяжение не нарушает.
170 500251
Расскажите мне про интересные теории на грани фантастики. Про космос, про возможные скрытые от взора планеты, про теории заговора в духе "у США были миссии на луну но там нашли корабль алиенов"
В детстве был впечатлен гипотетическими конструкциями как космический лифт и сфера Дайсона. Это как то пугает и одновременно восхищает.

В ZOG не отправляйте, там шизики.
171 500263
Почему цифры на циферблате микроволновки трясутся, когда я жую и смотрю на них?
172 500280
>>500263
Дрожат от страха.
173 500336
>>500251

>теории


Теория, маня, - это высшая форма научного знания.
А то, что

>на грани фантастики


это просто высеры какие-то.
174 500419
>>500241
Галактика же движется, если до нее свет долететь не может за это воемя. Ну и так вот
175 500443
>>500419
Никуда она не движется, она неподвижно впаяна в пространство. Ну то есть немножко движется, конечно, но мы сейчас не о собственном движении говорим.

В любом случае, расширение пространства это не движение. Могу ту самую аналогию с надуваемым воздушным шариком или растягиваемой резиновой лентой привести, только у нас будут не точки нарисованы на резине, а игрушечные машинки на ней стоять.

Допустим, максимальная скорость машинок — 3 м/с, расстояние между ними — 10 метров. Мы взяли и растянули ленту в два раза за одну секунду, расстояние между машинками увеличилось со скоростью 10 метров за секунду, но при этом они не только свою максимальную скорость не превысили, но и даже не двинулись с места.
176 500454
>>500443
Но ведь по телевизору сказали, что эфира не бывает, а пространство изотропно.
177 500493
Теоретически,

1. могут ли у земли вместо луны ( или слвместно с ней, раза в два меньшей по массе) быть кольца как у сатурна?

2. Может ли юпитер или сатурн располагаться настолько близко к земле, что его спутники (кольцо) было бы видно невооруженным взглядом, и если это возможно в теории, то повлияла бы гравитация любой из них на землю?
178 500499
>>500493
1. Вместо могут, хули б нет. Правда, хз могли ли бы они возникнуть естественным образом. Совместно с Луной - сомневаюсь. Думаю, Луна слишком массивная и быстро разогнала бы всю эту шушеру нахуй.

2. Запросто, особенно при максимальном сближении по орбитам. Марс с Земли отлично виден, так что помести на его орбиту Юпитер - и его спутники будут видны (Ганимед будет может раза в два тусклее нынешнего Марса). Гравитация бы как-то влияла, но вряд ли существенно. Это все в районе искажения орбит за миллионы лет.
179 500501
>>500493

>Chinese astronomical records report that in 365 BC, Gan De detected what might have been a moon of Jupiter, probably Ganymede, with the naked eye.[26][27]


Ганимед теоретически даже там, где он есть, можно увидеть, у него звездная величина до 4,38 доходит.
180 500507
>>500501
Не, ну я имею ввиду чтобы пять крупнейших спутников выглядели как вполне различимые на ночном небе звезды.
Или в случае сатурна прям можно было увидеть щель кассини.
Тогда они наверное вообще адово бы выглядели, как мини- луны
181 500543
>>500454

>эфира не бывает


Не бывает

>пространство изотропно


На масштабах более 400 МПк очень даже однородна, да
Как это на расширение-то влияет?
182 500547
Весной смотрел в телескоп на Марс, и показалось, что видна большая полярная шапка. Захотел проверить, так ли это, или просто искажение в телескопе, но не нашёл актуальных изображений планет. Есть ли ресурсы, где можно посмотреть, как в настоящий момент выглядят планеты?
183 500548
>>500547
Вотпрямщас - нет. А всякие космические агенства на своих сайтах выкладывают фотки с космических обсерваторий и телескопов. Загугли какой сейчас сезон на Марсе и смотри фотки с того же сезона.
184 500561
>>500547
В группах астрофотографии в вк постоянно выкладывают актуальные только что снятые фото планет.
Есть еще известная в астрономических кругах команда DS Astro - у них тоже полно постоянно фотографий планет, Луны и всего остального.

Если ты смотрел весной в прошлом году - в это время на Марсе уже бушевала пылевая буря на всю планету. Возможно ты и видел полярную шапку, но скорее всего это были ее отголоски, и без пылевой бури было бы лучше
185 500562
>>500561
Спасибо.
Смотрел этой весной, в апреле. Один из полюсов был ярко голубой
186 500570
>>500562
Почитай про Марс. Там действительно весьма ощутимо меняются полярные шапки по сезонам, пушо углекислый газ то намерзает, то испаряется. Еще и орбита довольно вытянутая, так что расстояние от Солнца заметно меняется.
187 500618
>>500547
На Марс можешь посмотреть тут: http://www.msss.com/msss_images/subject/weather_reports.html
188 500628
А как повлияло бы на землю наличие вместо луны кольца совокупной массой как луна? Сутки могли бы быть те же по продолжительности? В приполярных широтах было бы светло даже ночью?
Коимат холоднее?
189 500633
Возможно ли передавать данные в космосе на нормальных скоростях хотя бы несколько мегабит ? Про то что пинг высокий очень знаю, но почему нельзя большой канал сделать чтобы за секунду не жалкие десятки/сотни бит шли ?
190 500634
>>500628

>Сутки могли бы быть те же по продолжительности?


Ну при тех же начальных условиях, сутки были бы короче, потому что не было бы приливного замедления, как я понимаю. Сейчас именно Луна понемногу тормозит вращение Земли, за счет этого удаляясь. Кольца бы "тянули" со всех сторон примерно одинаково, значит приливных эффектов бы не было.

>В приполярных широтах было бы светло даже ночью?


Тут хз, но наверное да. Кольца массой с Луну обладали бы огромной поверхностью, поэтому бы отражали гораздо больше солнечного света.

>Коимат холоднее?


Не вижу, как бы это на климат серьезно повлияло. Разве что днем кольца бы не давали пройти части солнечного света, но подозреваю, что слишком малой части.
191 500635
>>500628
Ты охуел, даже у Сатурна кольца в разы легче Луны
>>500633
Ограничено скоростью света. Большой канал делать нецелесообразно, так как масса антенны и источников энергии, к тому же в космосе все равно нихуя не происходит, спешить с передачей некуда.
image.png171 Кб, 650x366
192 500651
лол это правда?
193 500654
>>500651
Если концентрация будет высокая, то да. В естественных метеоритах железо-никель и силикаты скучные в основном.
Сейчас есть японский стартап, который хочет делать что-то типа фейрверка в небе из химически концентрированных метеоров, но их тест кажется в конце года и посмотрим на результаты.
194 500658
>>500654
Но они же в основном каменные углеродные или железо-никелевые, откуда серьезная концентрация иных веществ? Как вообще может быть азот и кислород, даже кометы ледяные в основном
эту хуйню скинул тори бруно если че
195 500659
>>500654
А девочки-волшебницы планируются?
196 500662
>>500658

>откуда серьезная концентрация иных веществ?


Ниоткуда, перечитай пост.

>Как вообще может быть азот и кислород


с Титана сможешь наблюдать, если будут сгорать в его атмосфере.
sage 197 500663
Хотя в теории могут быть астероиды с высокой концентрацией магния, соды, кальция, если их выбило с поверхности тела, где они были сконцентрированы на поверхности. Типа если на Луне был разлита магний-лава из недр, она застыла, рядом врезался астероид в Луну, образовав кратер, магний-камень выкинуло на орбиту и он врезается в Землю - получишь фиолетовый хвост. Такое же говно может выбить грунт из Цереры и получишь содовый астероид, из Меркурия - кальциевый...
sage 198 500665
>>500663

> получишь голубоватый хвост


фикс
photo2019-08-1201-05-16.jpg122 Кб, 1080x1268
199 500700
мне вот что интересно - насколко толстой получится корка льда, если на земле будет так холодно, что промёрзнет, сжтжится, и перейдёт в твёрдую фазу атмосфера?
ну допустим на субсветовой скорости в солнечную систему влетела небольшая чёрныя дыра, и из-за гравитационных волнений нашу земляшу со скоростью больше второй, а то и третьей космической выкидывает прочь от солнца - берём в рассчёт, что с таокй силой, что к солнцу не вернётся. и что тогда? насолько быстро земля остынет? если какой-нибудь морксой исследовательный лайнер, находящийся в точке немо, решит пиздовать в санкт-петербургский порт, за сколько он приплывёт? успеет ли океан промёрзнуть? а успеет ли корабль сковать льдом? а какой толщины над ним будет лёд атмофсеры и будет ли атмосферный лёд такой толщины или там напертся пара метров от силы?
а если скорость покидания земляшой сраной солнечной вообще бическая, и землч уже через пару месяцев пролетит орбиту плутона, насколько быстро всё остынет и обратиться в лёд?
200 500750
>>500700
Не хочется разбирать тобою написанное, может кто другой подробней объяснит. Но это платина, и когда-то давно ИТТ выпавшую криоосадками атмосферу без коэффициента уплотнения насчитали слоем на 11 метров.
201 500755
Объясните, какого хуя при взрыве сверхновых всякие тяжёлые элементы оказываются выброшены наружу, а не коллапсируют вместе с ядром, они же сука тяжёлые, а рассеиваться должен только водород.
202 500757
>>500755
Во-первых, взрывы сверхновых несимметричны, поэтому же, кстати, они и двигаться начинают после взрыва. Во-вторых, в массивных звездах элементы тяжелее гелия расположены слоями, от ядра, и во время взрыва, синтез начинает идти по всему объему. ЕМНИП, чем тяжелее элементы в ядре звезды, тем быстрее проходят стадии синтеза и синтез железа занимает совсем короткий срок. После этого никто точно не знает, что происходит, но эти процессы за считанные минуты приводят звезду к коллапсу и давление с температурой подскакивают настолько, что синтез идет дальше, потому что сверхплотная материя ядра нейтронизируется и начинает срать нейтронами, которые и захватывают все, кому не лень.
203 500760
>>500755
Тяжелые элементы синтезируются во время самого взрыва, когда внешние оболочки звезды падают на ядро, сжимаются и загораются.
204 500762
>>500618
Спасибо, анон! Даже по датам видео разложены. Теперь я знаю, что мой маленький телескоп правильно всё показывал. Не ожидал, что шапка может быть такая здоровая, и Марс чуть ли не на треть голубой
15656366386070.jpg76 Кб, 576x768
205 500784
>>500750
а за какой срок планета с нашего сегодняшнего состоянии остынет до полностью ледяной корки?
206 500801
>>500700

>успеет ли океан промёрзнуть?


Сверху океаны замёрзнут за несколько суток, максимум неделя. Если совсем без солнечного света земляшка останется. А вот как там внутри на глубине будет это хз, там уже внутренняя геотермальная энергия подогревать какое-то время будет.
207 500803
Золотая пластинка вояджера. Там координаты основаны на пульсарах. Пульсары двигаются в галлактике. И как иные цивилизации будут угадывать, где находились пульсары которые они наблюдают, и как их соотнести с тем что нарисовано? Это проёб или здесь что-то более сложное?
208 500804
>>500633
догадайся сам
209 500806
хм
210 500808
>>500803
Забьют текущие координаты и векторы скоростей пульсаров в компудахтер с программой-симулякром, промотают симулякцию на тыщу лет назад и будут чекать каждый год положение пульсаров - если совпадает с пластинкой, то збс вот оно. Потом на этом основании вычислят текущее положение земляшки и ебанут по ней.
211 500812
>>500803
к тому моменту мы вымрем
212 500813
>>500808
Чому Саган решил, что это может сработать? Странно ожидать, что если где-то есть другой разум, то он думает как мы и проссыт, что это именно пульсары и вообще типа карта. Да я бы и сам не догадался, хотя я туповатый конечно.
другой хуй
213 500819
>>500813
Потому что разум, вышедший в космос, вполне способен к анализу чего-то ему неочевидного.
15408312893970.jpg5 Кб, 200x199
214 500822
>>500808

>Потом на этом основании вычислят текущее положение земляшки и ебанут по ней.

215 500841
>>500813
Пушо Саган - ефремодебил
Razmeryi-planet-i-zvezd.jpg56 Кб, 880x578
216 500877
А может ли быть жизнь на звездах и что из себя представляет их поверхность?
217 500881
>>500877
поверхности никакой нет, есть газ и плазма, сдерживаемые магнитным полем и гравитацией
изображение.png58 Кб, 1280x1024
218 500882
Я не пойму про точки лагранжа. Ну ок, с L1 понятно. Но с хуя ли в других точках корабль будет неподвижен? Взять хотя бы L2. Предмет в ней должен же упасть на планету, разве нет?
219 500883
>>500882
Он будет неподвижен не относительно Земли, а относительно системы Земля - Солнце. Вокруг Земли он будет вращаться, просто его орбитальный период будет равен орбитальному периоду Земли вокруг Солнца.
220 500887
>>500881
жизнь и колонизация Цефеи А или Большого Пса возможны?
221 500888
>>500882
нет. л2 значительно выше земли. объект там вращается по обычной орбите и должен отставать от земли так как высокие орбиты более мендленные, но земля немного тянет к себе, выравнивая скорость.
>>500883
бредишь
222 500889
Чем звезда по имени солнце отличается от газового гиганта юпитера? Кроме размера
223 500890
>>500889
массой, и как следствие наличием термоядерных реакций
224 500891
>>500877
Плазма слишком дохуя подвижна. Если из нее склепать элементы для построения тела, то их быстро распидорасит. Может где-то в глубине, где ебейшее давление и плазма кристаллизуется, возможно сделать устойчивое тело, подключить к нему сознание и жить. Но кто этим занимался? Ладно у нас - прилетел, терраформировал под нужные условия, разработал и населил всякими видами, улетел. А как ты будешь это делать внутре Солнышка? Тут нужно обладать сверхтехнологиями, граничащими с метафизикой, такое под силу, наверное, только внешним конструкторам миров, а не обитателям вселенной. Даже если они заложат жизнь в нескольких звездах - она не сможет самостоятельно распространятся по другим звездам, то есть делать это не целесообразно. Так что скорее всего в звездах никто не живет.
TheIceGiants.png45 Кб, 600x298
225 500892
Пикрелейтед зовется газовыми ледяными гигантами. Почему говорят об отсутствии поверхности у них, если лед уже поверхность?
226 500893
>>500892
При слове поверхность имеет в виду землю. Постоянно твёрдая поверхность то есть. Лёд не постоянен и на нём никогда ничего не вырастет.
227 500895
>>500893
Но на него же можно высадиться, а, учитывая температуры на планетах - передвигаться и построить ваакумное жильё, без ипотеки.
228 500897
>>500895
С таким же успехом можно высадиться на ядро юпитера
229 500898
>>500897
нет, там облака одни газовые с аммиаком, на ледяных гигантах же есть за атмосферой и мантией ледяная поверхность.
230 500911
>>500888
Все правильно он говорит. Отставать от Земли он не будет, потому что, внезапно, к "центру" его притягивает не только Солнце, но и Земля, так что орбитальная скорость будет подходящей. А вокруг Земли он будет вращаться с периодом один год, оставаясь при этом все время спрятанным Землей от Солнца. Это из той же оперы, как Луна, все время повернутая к Земле одной стороной, но все же вращающаяся вокруг своей оси.
231 500914
>>500898
Нет там никакой ледяной поверхности.

>Ледяная оболочка фактически не является ледяной в общепринятом смысле этого слова, так как состоит из горячей и плотной жидкости, являющейся смесью воды, аммиака и метана[9][58]. Эту жидкость, обладающую высокой электропроводностью, иногда называют «океаном водного аммиака»[59].


>Жидкая внутренняя структура означает, что у Урана нет никакой твёрдой поверхности, так как газообразная атмосфера плавно переходит в жидкие слои[9]. Однако, ради удобства за «поверхность» было решено условно принять сплющенный сфероид вращения, где давление равно 1 бару.

232 500917
>>500877
Нет конечно, там же гигантские температуры и давление, мб только на потухших белых карланах
233 500919
>>500917
типо альдебарана и Ригеля?
234 500922
>>500919
Альдебаран во всю горит, я имею ввиду совсем потухшие
235 500924
>>500919
коричневые карлики.. в теории возможно
236 500930
А почему в атмосфере газовых гигантов, атмосфера плавно перетекает в ядро, а в атмосфере земли граница двух сред явственно заметна?
237 500931
>>500930
из-за веса
238 501026
>>500877
Одним из основных критериев существования жизни служит химическое разнообразие. Химическое разнообразие требует некоторого температурного окна как минимум. Вопрос уровня возможно ли нормальное функционирование городской инфраструктуры с движением автомобилей на гоночных скоростях формулы-1? В текущем виде нет, это противоречит законам физики. Какие-то концептуальные проекты изъебнуть можно, но это будет именно изъебством, а не естественным путем оптимизации и прогресса. Изъебства вселенной не свойственны.
239 501039
>>500898
так газы при низкой температуре превращ в жидкость
240 501042
>>500922
Совсем потухших пока что не бывает, за пару миллиардов лет ничего не остынет. Обращайтесь в 20000000018 году.
241 501143
>>501039
от довления зависит во что они превращаются.
242 501147
>>500930

>атмосфера плавно перетекает в ядро


откуда инфа?
243 501164
>>501147
ПРЕДПОЛОЖЕНИЯ и МОДЕЛИ. На большее хоть когда-нибудь рассчитывать вряд ли стоит, ни один материал не выдержит того, чтобы хотя бы газовый слой полностью пройти до границы сверхкритического перехода.
244 501200
>>501164
А радаром чо нельзя просветить?
245 501201
>>501042
20000000019
246 501204
>>501164
на венеру вроде запускали, на венере давление разве меньше чем на юпитере? Я понимаю что на юпитере ебейшие ветра, но всё же
247 501210
что такое экзопланеты?
248 501212
>>501210
Экзопланета (др.-греч. ἔξω, exō — вне, снаружи), или внесолнечная планета, — планета, находящаяся вне Солнечной системы.
249 501213
>>501210
это все планеты названия которых ты не знаешь
250 501217
>>501212
откуда тогда у астрономов данные по их массе, составу, возрасту.
я просто слушаю когда (люблю ролики про Астрономию на ютуб) то ахуеваю. Просто с потолка же похоже на ахуительные истории.
251 501218
>>501210
гипотетические планеты вращающиеся вокруг других звёзд, не Солнца. Никто их не видел и ещё очень долго не увидит, даже в самые мощнейшие телескопы, только предполагают их существование
252 501220
>>501218
я могу предполагать существование ламповых тян.
Как на серьезных щщах можно такое явление вообще обсуждать?
253 501221
>>501217
примерно так почувствовали на основании орбит и прочего
800px-StructureofJovianatmosphere-rus.svg.png48 Кб, 800x600
254 501222
>>501204
Давление на твёрдой поверхности венеры ~93 бар.(алсо там углекислота уже в сверхкритическом состоянии) Это 250 км атмосферы.
Максимально известные данные о юпитере это 132 км. Там давление достигает вот таких вот значений, если верить картинке.
255 501223
>>501217
когда планета пролетает перед звездой её становитсявидно, точнее черную точку которая загораживает часть солнца.. Ну и путем алхимии и прочих расчетов примерно высчитывают размер планеты, опять же всё это не на 100%
256 501224
>>501222
1000 мбар это 1 бар!?
258 501226
>>501224
1 миллион бар
259 501227
image.png183 Кб, 524x383
[b]Наблюдение за звездами - опасное хобби.[/b] Любитель 260 501310
Наблюдение за звездами - опасное хобби.
Ты уезжаешь в жопень где потемнее с огромным телескопом и молишься, чтобы никакой бандит не прошел рядом и не убил/ограбил тебя.

У меня вопрос. Как вы защищаете себя в 2,3,4 ночи в глуши? Были опасные случаи? Кто умер
261 501313
>>501310
Кто умер, тот уже не напишет.
262 501314
>>501313
это да, но мне то умирать не хочется. Что делать чтобы не стать жертвой бандита? сам что делаешь?
263 501315
>>501310
Я умер. Ночью сидел боялся бандитов, очень сильно боялся, весь в напряжении такой сидел, про девяностые вспоминал, пот в окуляр телескопа стекал, очень боялся в общем. Потом я непроизвольно пукнул и умер от разрыва сердца.
264 501317
>>501315
твоя юмореска конечно забавна, но она лишь смеется над моей проблемой и не решает ее. Или же ты хочешь сказать с помощью этой шутки юмора, что в наше время ничего бояться не стоит, потому что 90 прошли и можно спокойно идти и делать что хочешь в любое время, а я запуганный шизик?
265 501319
>>501317

>в наше время ничего бояться не стоит, потому что 90 прошли и можно спокойно идти и делать что хочешь в любое время, а я запуганный шизик?


Схватываешь на лету, братан.
Ну баллончик перцовый купи, и валерьянки пузырёк.
266 501320
>>501319
а как ты объяснишь такой случай:
шли мы как-то с друзьями по лесу, в километрах в 2-3 от входа в него собственно, и натыкаемся на совсем небольшое озеро, на другой стороне которого был мужик с собакой. Сначала все было хорошо: мы посмотрели друг на друга и пошли дальше прогуливаться, как вдруг с его стороны раздается злостный крик "ААААРРРРРРРРР"!!!. Мы тогда знатно испугались и покинули лес. Он был явно агрессивен. Благо я был на легке и быстро убежал, а вдруг со мной был громоздкий телескоп? Онбы догнал меня и отобрал его :(
267 501325
>>501320
Помню гулял по лесу с собакой, смотрю - какие-то аутисты из леса вышли. Ну я на них внимания не обратил особого, дальше пошёл. Вдруг как об корягу мизинцем ебанусь, аж закричал. А эти два по съёбам дали, я даже успел заметить что у одного пятно коричневое на штанах появилось.
268 501327
>>501325
не ну даешь юмор в массы)) но ситуация действительно вызывает ужас. Мы одни и кто знает что было бы если не наши быстрые ноги.
269 501329
Натыкался на охотников с ружьями ночью когда выезжал за 70км от города смотреть дипскай с другом, в дичайших пустых полях и лесополосах вокруг с 0 количеством людей и домов на километры вокруг.
Они поинтересовались что за хуевина огромная и попросили посмотреть. А потом предложили пострелять из ружья.
Плюс еще наша тачка стояла рядом. Как нехуй было убить и украсть все добро.

Но как то все разы все спокойно проходило когда в дикой степной глуши встречались случайные незнакомцы или машины, где казалось их вообще быть не должно, потому что ты единственный шизик ночью в этом месте выехал посмотреть звезды в телескоп.

Ничего страшного нет. Ночью все и каждый человек выглядит как злонамерянный психопат.
Можно носить с собой несколько баллончиков и держать в пределах секундной досягаемости.

А еще лучше сходить к психологу и избавиться от этого страха и жить спокойно.
270 501331
>>501320

>раздается злостный крик "ААААРРРРРРРРР"!!!


Ahoy Maties!
271 501338
>>497985 (OP)
Почему ученые ищут сигналы во всплесках? Ведь если другие формы жизни exists, то они уже колонизировали космос и повсюду, то есть этот сигнал постоянный, а не всплеск.
272 501349
>>501338
всплески слушают с целью изучения их источников, а не поиска внеземной жизни
273 501384
>>501338

>формы exists


Лучше б ты не выебывался позорился.
274 501385
275 501411
Украина так зависит от российского газа, почему бы им не колонизировать газовый гигант и качать возить газ оттуда, там все равно запасы такие, которые человечество и Земля никогда в жизни не израсходуют.
Ответ, нужны ответы, помимо того, что хохлы дегенераты
276 501417
>>501411
Это крайне дорого, туда сюда ракеты пулять, тем более с Юпитера ее хуй запустишь
277 501418
>>501417
кто тебе сказал?
278 501420
>>501200
Горячий газ непрозрачен для излучения. Да и от чего там радиоволнам отражаться?
279 501427
>>501411
Если уж разводить мрии об альтернативной хайтековой Украине, то самый охуенный вариант - фармить метангидраты в Чёрном море, где их просто овердохуя. (А общемировые запасы метангидратов превышают запасы нефти в сотни раз.)

Это намного проще добычи ресурсов в космосе, но пока таких технологий ни у кого нет. Активнее всего в ту сторону копают японцы (вокруг Японии метангидратов тоже овердохуя). Если самураи родят технологию - они нахуй перевернут весь рынок.
280 501434
>>497985 (OP)
А правда что на Марсе есть портал в ад?
1565936815821.jpg34 Кб, 480x300
281 501436
>>501434
Да, вот фото с марсохода.
282 501437
>>500930
там сначала как-бы газовая атмосфера, а затем очень глубокий океан из жидкого газа, затем газ уже переходит в твёрдое ледяное состояние.

Так-что газовые гиганты это скорее планеты океаны из жидкого газа вместо воды.
image.png67 Кб, 336x280
283 501450
>>501437

>жидкий газ

284 501452
>>501450
Он пиндос просто.
286 501484
Пацаны, как перевезти на марс 200 млрд тонн кислорода?
287 501486
>>501484
Баржами.
sage 288 501489
>>501484
Нужно нанять 20 млрд газонюхов, чтобы они таскали газ из очка Земли и выдыхали его на Марсе.
289 501490
>>501434

> А правда что на Марсе есть портал в ад?



Если и был - то всё!
Мы позаботились об этом:

> «Святая Русь будущего — это воцерковленный народ с мощной армией и промышленностью, православными компьютерами. Марс будет канонической территорией РПЦ, а сектантов мы туда не пустим».



https://www.kommersant.ru/doc/2857941
290 501498
>>501484
Зачем ? Чтобы его опять сдуло к хуям ?
291 501544
Всем привет
не секрет что первым колонистам Марса придется совершать самоубийство к окончанию вахты.
Вопрос: Как поведет себя огнестрельное оружие на марсе? (пистолет) Смогут ли они застрелится?
292 501548
>>501544

>Как поведет себя огнестрельное оружие на марсе?


Так же, как на Земле. Ну бабахнет намного тише. Огнестрел даже в вакууме работает, если чо - порох спокойно горит и без доступа кислорода.
293 501550
>>501548
Как это горение без кислорода или я что-то прогуливал в школе?
А что на скорость пули данные марса не влияют? Там же нет воздуха для сопротивления, может быстрее полетит? Тем более и притяжения мало.
294 501559
>>501550

>Как это горение без кислорода или я что-то прогуливал в школе?


Ну ты кек, порох сам по себе горючее + окислитель

>А что на скорость пули данные марса не влияют?


Ну влияют конечно, пуля пролетит дальше чем на земле, но когда стреляешь себе в голову этим можно и пренебречь.
295 501562
>>501310
Хуй с ними с бандитами. Холодно ночью шо пиздец! Вот холода надо бояться. Даже летом.
296 501565
>>501544

>не секрет что первым колонистам Марса придется совершать самоубийство к окончанию вахты.


Это что за шизофрения?
297 501567
>>501565
Это он забайтил чтобы быстрее ответили или чтобы потралировать, а ты и попался
изображение.png271 Кб, 451x320
298 501570
299 501572
>>501565
>>501567
Лолка, если 2 года на вахте там будешь, то второй мозг вырастет от радиации в разных частях тела, так что особо выхода нет.

или ты думаешь туда-сюда каждый месяц гонять будут чтоли?
300 501576
>>501572
Действительно, зачем делать защиту от радиации и возвратные корабли, у нас же очередь стоит из квалифицированных специалистов, желающих отхуярить два года на другой планете и потом застрелиться, а краткосрочные экспедиции с кораблями возврата ожидающими на орбитальной станции были запрещены на законодательном и квантово-волновом уровнях ещё в 618 году н.э..
Поляков говорят после 437 суток на Мире вообще застрелился из-за того что у него от радиации хуй на лбу вырос, а на Земле потом просто бионическую куклу журналистам показывали.
Стреляных полярников так вообще штабелями укладывают под опорами научных баз, для изоляции.
301 501577
Антош, а как рассчитать на какие широты зимой падала бы тень от колец, будь они у земли?
И второй вопрос, будь это кольцо маленьким по массе - оно может быть полупрозрачным, в смысле, худо бедно но пропускать свет. Материал плюс минус тот же что и луна
302 501578
>>501576
Может их детей освободят от налогов.
303 501582
На какой из планет солнечной системы ты потеряешь сознание (умрешь) позже всего?
допустим, наберешь воздуха на 5 минут для дыхания с собой.
001.jpg1 Мб, 700x1006
304 501583
>>501576
Это ты ещё наших специалистов по разделению ступеней не видел.
305 501584
>>501582
На Марсе я думаю.
306 501585
>>501584
Там можно найти место с нормальной температурой? на горе Олимп? или в тени?
307 501590
>>501577

>как рассчитать на какие широты зимой падала бы тень от колец, будь они у земли?


Зависит от радиуса колец. Причем очень сильно, вплоть до того, что мимо "падала" бы.

>И второй вопрос, будь это кольцо маленьким по массе - оно может быть полупрозрачным


Думаю, что не просто может, а прям таки обязано. Иначе оно слишком тяжелое будет.
308 501592
>>501590
Спасибо. Добра
309 501595
>>501592
Ты бы подождал, может еще кто-нибудь что скажет. А то так сразу на веру принял.
310 501600
>>501595
Дак я никуда не ухожу. Но человек ответил - почему 6е поблагодарить? Добродвач же
311 501605
>>501582
На Земле.
1565998252906.png121 Кб, 1440x2712
312 501616
Может ли черная дыра иметь на своей орбите одновременно планету, с которой эта черная дыра будет видна невооружённым глазом (как мы видим Луну), и одновременно звезду, которая будет поддерживать жизнь на планете?
Может ли или должна эта чд быть при этом супермассивной?
313 501617
>>501616
Всё что оказывается на орбите чд, засасывается в чд. Так что лучше не приближайся.
314 501618
>>501616

> Может ли черная дыра иметь на своей орбите одновременно планету, с которой эта черная дыра будет видна невооружённым глазом (как мы видим Луну), и одновременно звезду, которая будет поддерживать жизнь на планете?


Черная дыра будет видна разве что за счет эффекта линзирования , а так если она не поглотила звезду или еще какое вещество и не размазала его аккреционным диском ничего видно не будет, вполне можно вращаться вокруг черной дыры если ты достаточно далеко от горизонта

> Может ли или должна эта чд быть при этом супермассивной?


Может, но жизнь там врядли тогда будет возможна центральные галактические дыры всегда активные она спалит к хуям твою планету излучением или выплюнет джет вообще
315 501619
Есть в треде у кого есть телескопы и опытные их пользователи?
У меня вопрос по поводу монтировок. Кажется я придумал новый легкий, маленький подтип монтировки для определенного диапазона габаритов труб и типов наблюдений, который почему то нигде не используется. Интересно почему и говно ли я придумал.

Сам обладатель 10’’ добсона и 114eq. И часто сталкиваюсь с проблемой мобильности телескопов. Особенно 114мм, труба которого весит пару кило, а монти занимает в 3 раза больше места и массы чем сама труба, что заебывает.
316 501620
>>501616

> Может ли или должна эта чд быть при этом супермассивной?


Так-то наша солнечная система и так вращается вокруг супермассивной чд в центре млечного пути
317 501630
>>501620
Не вокруг нее, а вокруг галактического центра масс. Вклад собственно СМЧД ничтожен, от её отсутствия ничего бы не изменилось.
318 501633
>>501582

>На какой из планет солнечной системы ты потеряешь сознание (умрешь) позже всего?


Если у тебя есть дирижабль, то внезапно Венера. Теоретически можно хоть несколько суток без скафандра в одних трусах протянуть, пока от голода не подохнешь.

>Несмотря на экстремальные условия на поверхности планеты, на высоте 50—65 км атмосферное давление и температура практически такие же, как на поверхности Земли. Это делает верхние слои атмосферы Венеры наиболее похожими на земные в Солнечной системе (причем даже больше, чем на поверхности Марса). Из-за сходства давления и температуры, а также того факта, что воздух для дыхания (20,9476 % кислорода, 78,084 % азота) на Венере является поднимающимся газом (так же, как гелий является поднимающимся газом на Земле), верхние слои атмосферы были предложены учёными в качестве подходящего места для исследования и колонизации[15].

319 501634
>>501633
В одних трусах не получится, сернокислотные облака на кожу плохо влияют.
320 501635
>>501619

> Есть в треде у кого есть телескопы и опытные их пользователи?


Здесь же где-то был телескпоный тред. Спроси там.
321 501642
>>501590
Вообще у Луны плотность высокая сравнительно, кольцо будет близко к Земле, должна тень попадать.

И космические полеты любые по пизде.
322 501759
1. Пиздошим термоядерными бомбами полярные шапки марса
2. Концентрация углексилого газа, водяных паров, аммиака, азота увеличивается
3. За счет парникового эффекта нагревается грунт
4. Высвобождается еще больше водяных паров из грунта
5. ...
6. PROFIT!

мм?
323 501764
>>501759
То что ты растопил, снова выпадет на поверхность и замерзнет. Или вы РЕАЛЬНО думаете, что газ такой "Фу, зашквар на эту шапку оседать! Тут бомбардировка была! Будем вечно теперь в атмосфере крутиться и разогревать, работать без продыху" ?
324 501766
>>501764

>То что ты растопил, снова выпадет на поверхность и замерзнет


Как оно выпадет, если температура поднимется за счет парникового эффекта?
325 501767
>>501766
>>501764
Алсо первое время газ будет нагрет от энергии взрывов. Потом уже через время начнет действовать парниковый эффект. Просто надо жахнуть сразу много и одновременно.
326 501768
>>501759
Бомб не хватит, даже царь бомбы в планетарном масштабе хуйня, уж лучше тогда астероиды кидать и даже если ты запилишь атмосферу ее один хуй сдует солнечный ветер у тебя же не магнитного поля, ни сильной гравитации нет
327 501769
>>501766

>поднимется за счет парникового эффекта?


Пруф? Кроме отсылки к авторитету, хотя Маск даже не авторитет так как он нихуя про климатологию Марса не знает и не консультировался со специалистами, прежде чем пиздеть. Давай, запости статью или вычисление, где как заключение будет "мы можем стимулировать парниковый эффект".
328 501770
>>501768

>Бомб не хватит


Хватит, если конечно человечество задастся такой целью. Дейтерий не такой дорогостоящий чтобы париться из-за этого. Вся проблема в политике и международных договорах.
Кидать астероиды тот еще бред, тебе придется веками ждать, пока астероид прилетит в марс, потому что мы не сможем значительно изменить его траекторию. И даже через век мы не сможем точно спрогнозировать его орбиту.
329 501771
>>501769
Нихуя не понял причем тут маск, но вроде еще совки изучали возможность парникового эффекта на марсе
14182232212031861853.jpg409 Кб, 1600x1184
330 501773
>>501771

> еще совки изучали

1566067395580.png1,3 Мб, 1920x1080
331 501788
>>497985 (OP)
Поясните насчет колец Сатурна и других газовых гигантов. Они и вправду состоят из мелких камушков, а не из огромных астероидов, как показывают в кино и играх?
332 501789
>>501768
Солнечный ветер долго сдувать будет.
333 501790
>>501788
частицы в кольцах сатурна от 1см до 10м, иногда могут попадаться комки, но огромные комки долго не живут, так как взаимодействуют с пылью и могут распадаться. Некоторые внешние кольца могут состоять из более мелких частиц. Про кольца других газовых гигантов известно меньше, у Юпитера мелкая пыль там, у Урана и Нептуна хуй знает, гипотезы лишь.
334 501791
>>501790

> частицы в кольцах сатурна от 1см до 10м


И сколько это в среднем? По 5 м?
1566069808850.jpg16 Кб, 620x462
335 501799
>>501791
Бамп вопросу. На каком расстоянии эти комки друг от друга?
336 501800
>>501799
От 1 км и более.
337 501803
>>501800
То есть если залететь в кольцо, то нихуя не увидишь, ни одного обломка?
338 501806
Да большинство этих колец состоят из мелкой пыли, как пылевые и пустынные бури у нас. Мелкая мелкая взвесь частичек. Чем дальше от Сатурна - тем мельче пыль и менее плотная.

А камушки несколько см и м тоже есть - из них состоят более плотные яркие кольца ближе к Сатурну.

Расстояние между камушками естественно меньше 1км, как если взять грунт лопатой и подкинуть вверх.

Кассини например пролетал сквозь одно кольцо и фиксировал микрофонами микро удары по корпусу.

Почитайте в инете инфу, епта, ее куча на той же вики.
339 501808
>>501806
А крупные камни размером в несколько метров как часто встречаются?
bliss.jpg2,4 Мб, 4510x3627
340 501810
Хочу стрелять из гигантской пушки в космос болванками с топливом и всякой небьющейся нехрупкой аппаратурой. Болванки, долетя до космоса, будут на самом простом и дешевом движке развивать горизонтальную скорость для выхода на орбиту. Потом на обычной ракете будет доставлена хреновина, которая будет состыковываться с этими болванками, собирая их в кучу, чтобы использовать их для строительства йоба-станции для покорения солнечной системы.
Какие подводные?
341 501811
>>501810
Дорого. Плюс ты все топливо потратишь на стыковку. Двигаться по орбите тоже затратно.
342 501812
>>501811

>Двигаться по орбите тоже затратно


Наркоман?
343 501813
>>501812
Не по орбите, а находясь на орбите. Я проебался с термином. Короче двигаться в разных направлениях, пока ты летишь по орбите.
344 501815
>>501813
Наклонения менять что ли? И если да, то нахуя?
345 501816
>>501759
https://m.habr.com/ru/post/384165/
Тддр надо 55к царь бiмб чтобы растопить шапки, если повезёт то это увеличит давление до 5% от земного.
346 501817
>>501815
Ну грубо говоря вывел ты одну болванку на орбиту, а потом другую на ту же. Но они то в разных точках орбиты окажутся, то есть одна еще должна будет догонять другую. И на это тоже нужно топливо, в зависимости от дистанции.
347 501819
>>501817
Нюфааааань, иди в КСП тред.
348 501820
>>501819
Ты если газанешь на орбите, у тебя не скорость увеличется, а орбита вытянется.
349 501821
>>501820
Смотря в какую сторону вектор газования будет направлен.
350 501822
>>501821
Но факт в том, что для обычного гомановского перехода много топлива не нужно. Метров сто дельты хватит, если у нас есть несколько суток времени.
351 501823
>>501822
Нам не на другую орбиту перейти надо, а догнать другую болванку на этой же орбите.
352 501825
>>501823
Для этого переходят на другую, более быструю или медленную орбиту, мань. А потом возвращаются в нужный момент.
353 501826
>>501823
БЛЯДЬ! Ты орбитальную механику знаешь вообще? Нельзя так взять и "догнать" по той же орбите, у них скорости сука одинаковые, а если скорость поменять, то это будет другая орбита! Левантовского шуруй читать, и пока не прочитаешь, не пиши сюда.
354 501827
>>501825
Ну на эти маневры же нужно топливо. Да и просчитать все нужно точно.
>>501826
А если, скажем, болванки летят друг от друга в километре, то тоже нельзя взять и догнать?
355 501830
>>501827
Если в километре, то тут уже можно легонько газануть в направлении целевого объекта, но орбита, по факту, все равно поменяется, просто на таких расстояниях замкнутость орбиты очень слабо влияет и вектор скорости просто не успевает сместиться земным тяготением достаточно сильно. К МКС так и стыкуются, потихоньку жгут гидразин в направлении станции, а потом так же плавно тормозят.
356 501832
>>501827

>Ну на эти маневры же нужно топливо


Нужно

>Да и просчитать все нужно точно


Нет, ты все таки "Механику космического полета" за авторством Владимира Исааковича прочитай. Это подсчеты можно на листочке тетрадном сделать.
357 501833
>>501830
А если немножко пшикнуть гидразина, повернувшись к целевому объекту, то ты ведь изменишь свою относительную скорость по отношению к нему? Ну то есть начнешь потихоньку приближаться, пусть и со скоростью миллиметр в час?
358 501835
>>501833
Именно так. Но не забывай, что в этом случае у вас с ним вектора скорости, почти сонаправлены. Но по миллиметру в час все равно не получится, потому что ты все равно начнешь от него отдаляться потом.
359 501836
>>501835
Понял. Проще ебашить гидразин непрерывно, пока не подлетишь близко, а потом резко тормознуть.
360 501837
>>501836
Нет, не проще. Во-первых, это небезопасно, во-вторых, скорости около метра в секунду хватит, чтобы успеть до того, как гравитация растащит вас по собственным орбитам (Союзы вообще при стыковке к шлюзу со скоростью в сантиметры в секунду летят), а в-третьих, это слишком затратно по топливу. А рабочее тело в космосе - на вес золота, по крайней мере, пока людишки ТЯРД не изобрели.
361 501845
>>501837
Да у людишек просто опыта мало пока, а потом начнут летать на полной скорости, педаль в пол, потом так же резко тормозить.
362 501847
>>501845
Формула Циолковского с тобой несогласна.
363 501851
>>501847
Жлоб. Топлива жалко ему.
364 501854
>>501851

>Тапку в пол, тапку в пол!


>А почему не хватило топлива на снижение перигея ниже 100 км?

365 501855
>>501854
Ионными снизишь, если вдруг закончится.
366 501856
>>501855

>Ионники на банке с огурцами


Знаешь, я передумал. Не ходи в КСП тред больше никогда.
all.gif470 Кб, 267x200
367 501863
Почему частички гравитациисвета улетая от ЮпитераСолнца не промахиваются мимо меня? Неужели их там много излучается с единицы площади и с таким астрономически маленьким углом рассеивания? Как такое возможно?

Когда я смотрю в небо на далекую звезду из звездной системы разумных жуков, я её не вижу, потому что частички света рассеиваются и не доходят до моих глаз ВООБЩЕ? Или их недостаточно МНОГО долетает до зрачка?
Частички света энергию не теряют ни в зависимости от пройденного расстояния, ни от времени?
Рассеивание частичек света подчиняется нормальному распределению?
Гравитация воздействует точно так же точечно, как и свет? Частички гравитации от планеты разумных жуков могут пролететь мимо меня? А если они попадут, они непосредственно взаимодействуют только в ма-аленькой точке попадания, а не со всем моим телом?
Если я могу закрыться от частичек света заслоняясь от или отклоняя их, можно ли точно так же поступить с гравитационными?
368 501867
>>501863

>Почему частички света улетая от Солнца не промахиваются мимо меня? Неужели их там много излучается с единицы площади и с таким астрономически маленьким углом рассеивания? Как такое возможно?


Много излучается, да, порядка 10^45 фотонов в секунду. Плюс Солнце довольно близко по астрономическим меркам.

>Когда я смотрю в небо на далекую звезду из звездной системы разумных жуков, я её не вижу, потому что частички света рассеиваются и не доходят до моих глаз ВООБЩЕ? Или их недостаточно МНОГО долетает до зрачка?


Что-то доходит, телескопы же видят эти системы, просто глаз не телескоп и не может накапливать свет в течение длительного времени. Минимальный порог порядка нескольких десятков фотонов в секунду, при меньшей частоте просто фоторецептор не активируется и сигнал по нейрону в мозг не идет, фотоны.

>Частички света энергию не теряют ни в зависимости от пройденного расстояния, ни от времени?


Во-первых, для фотона это одно и то же, он всегда с одной скоростью движется. Во-вторых, теряют, Вселенная расширяется и длина волны летящего по расширяющемуся пространству фотона тоже слегка растет. Но этот эффект заметен только на масштабах в миллиарды световых лет.

>Рассеивание частичек света подчиняется нормальному распределению?


Нет, формуле Планка.

>Гравитация воздействует точно так же точечно, как и свет? Частички гравитации от планеты разумных жуков могут пролететь мимо меня? А если они попадут, они непосредственно взаимодействуют только в ма-аленькой точке попадания, а не со всем моим телом?


Решил украсть мою нобелевку за открытие гравитонов, пидарас?
369 501868
>>501867

>фоторецептор не активируется и сигнал по нейрону в мозг не идет, фотоны впустую пропадают.


Фикс
370 501887
>>501816
Если увеличить мощность в 10 раз (что вполне реально), то уже надо всего лишь 5к бомб. Что тоже вполне реально. Илон маск вон хочет миллион тон грузов перевезти на марс.
371 501894
>>501887

>5к бомб


На самом деле гораздо больше, в тех расчетах предполагается что вся энергия бомбы тратится на испарение газа, что невозможно. Но в любом случае, даже если испарить весь со2 и воду что есть на Марсе, то в результате давление белеет будет в разы меньше чем на Эвересте.
372 501896
>>501894
И где тогда воздух брать для марса??? А ведь основой атмосферы должен быть азот, а его там нету, и хз где его достать.
373 501897
>>501894
1. кроме со2 и воды есть и другие газы
2. обычный грунт марса содержит довольно много воды. если поднять температуру, то она начнет испаряться.
374 501898
>>501897
А зачем тебе атмосфера из воды?
375 501899
Потому что пусть лучше будущие марсиане ходят с сухопутными аквалангами, а не в громоздких скафандрах, которые ничем от Аполлоновских не отличаются. Решить проблему с давлением - это даже не полдела, это процентов восемьдесят дела.
376 501902
>>501896
Или вулканы или брать извне, третьего не дано.
377 501907
>>501896
Нужно пидорить грунт. закапывать термоядерные бомбы на 100 метров и взрывать. Там и азот найдется и вода и много чего еще.
15655160172180.mp413,8 Мб, mp4,
640x360, 4:15
Правду вам в тред умный 378 501908
Правду вам в тред
image.png13 Кб, 379x323
умный 379 501909
А вы не думали что нынешние люди лишь фотоновая тень во времени от оригинала?
380 501912
>>501902
Вулканы на марсе потухли давно.
381 501913
>>501907
Откуда на 100 метров азот?
382 501914
>>501909

>МААААМ, неси таблетки, дед опять за старое

383 501925
>>501914
так же столетия назад говорили о тех кто считал землю круглой...
384 501928
>>501907
Выкопать несколько десятков тысяч стометровых дыр даже на земле очень дорого
385 501930
Знатоки пространства и времени, хотел задать возможно тупой вопрос, но все же тред для этого и создан , так вот зависит ли течение времени только от скорости которое развивает тело и вообще как это работает? То есть известно, что чем быстрее разгонится какой нибудь гипотетический звездолет тем медленнее там будет идти время по сравнению с Землей, но ведь и у других планет есть своя скорость вращения вокруг звезды, звезды соответственно вокруг центра галактики , то есть означает ли все это, что время на какой нибудь планете в галактике Андромеды будет идти по другому чем у нас ?
386 501931
>>501928
Можно в одну дырку скидывать бомбы друг за другом
387 501933
388 501949
Сейчас по дискавери была передача про искусственные алмазы, в конце мужик добавил, что астрономами(с) обнаружена планеты в несколько раз больше Земли и предполагается, что она на треть состоит из чистого алмаза. Это какая-то гипотетическая байка или реальный астрономический объект или класс?
389 501958
>>501931
А что вообще будет при таком раскладе? Вот если бы в кольскую сверхглубокую такими макаром бомбы запускали, что было бы?
-3875723.jpeg822 Кб, 1200x703
390 501960
Что будет если в открытом космосе, в пределах зоны златовласки - жидкой воды, разлить гигантскую каплю воды, радиусом в 1 км, она замерзнет или испарится? Или покроется тонкой коркой льда, а внутри можно будет плавать как в аквариуме? Можно ли будет там создать подводную экосистему?
391 501966
>>501958
Кстати да, тогда же угорал совок по мирным ядерным взрывам , хули не взорвали бомбы после того как застряло бурение
392 501968
>>501949
Да, есть такая экзопланета. 55 Cnc, 40 световвх лет, в созвездии Рака.
393 501970
>>501960
Сначала вскипит, потом остынет до маленьких льдинок
394 501975
>>501960
Нет гравитации -> нет давления -> жидкость моментально выкипает к хуям.

Вот если бы капля радиусом хотя бы в 3000км, тогда заебись будет. Только это уже не капля, а целая планета будет.
395 501976
>>501966
Потому что совку уже пиздос пришел когда кольскую копали.
396 501979
>>501925

>Эратосфен


>276 год до н.э.— 194 год до н.э.


Дооооо
397 501980
>>501975
Тогда ее солнце выдует, как комету
398 501981
>>501979
-МАААМ, неси таблетки, Эратосфен опять за старое!
-Так их не изобрели ещё
-CYKA BLYAT
399 501999
>>501925
Со времен древней Греции у образованных людей не было никаких сомнений о форме Земли.

Фейк о том, что якобы до плаваний Колумба люди считали Землю плоской, выдуман в 19 веке, на самом деле никто так не считал. Более того, Колумба считали шизиком и отказывали в финансировании как раз потому, что все слишком хорошо знали форму и размер Земли, и что до Индонезии западным путем нихуя не 3-4 тысячи километров, а 20. Экипаж бы сдох в море от голода, если бы ему по дороге случайно не попался другой континент.
400 502016
>>501999
Ты Колумба с Магелланом перепутал кажись.
401 502017
>>501930

>То есть известно, что чем быстрее разгонится какой нибудь гипотетический звездолет тем медленнее там будет идти время по сравнению с Землей


Не совсем "по сравнению". Если смотреть с Земли на быстролетящий звездолет, то на нем часы будут идти медленнее, чем у нас. Но если с борта этого звездолета смотреть на Землю, то часы на Земле будут идти медленнее, чем у космонавта. Такие дела.
402 502018
>>502017
Что значит "смотреть"? Ты нихуя не увидишь при таких скоростях и расстояниях.
403 502019
>>501908
Кульно, что это?
>>501912
Последний раз олимпус извергался 2 миллиона лет назад. Как по мне это не называется потухнуть.

Во-первых тераформируй сначала сахару и антарктиду.
Во-вторых научись избавятся от нейтронного облучения.
В 3 научись дастовлять на орбиту марса миллионы тонн груза.
в 4 изменить траекторию астероида вполне по силам и сильно не надо его двигать, а за 3-4 витка его орбита не станет сильно непредсказуемой. А взрыв от метеорита не фонит радиацией и мощней.
В 5. Взрыв какой бы силы он ни был, и какой бы источник ни имела, не сделает атмосферу более тёплой. Он сделает температуру более тёплой в месте удара. Температура быстро перемешается, уйдёт на вибрации. А газ даже если и испарится не успеет сделать потепления ввиду быстрого остывания.
В 6 взрыв царь бомбы не растопил новую землю.

В 7 и самом главном. Зачем тебе это надо? Что вас так тянет в гравитационные ямы?
404 502020
>>502018
Ну бля, представь, что увидел. Без точки зрения, то есть системы отсчета, связанной с наблюдателем, сравнения смысла не имеют.
405 502023
>>502019

> Что вас так тянет в гравитационные ямы?


Невесомость негативно влияет на человеческий организм. Если мы будем жить в невесомости - мы превратимся в уродцев.
406 502024
>>502020
Типа смотришь с Земли в телескоп на часы, которые на борту звездолета?
407 502027
>>502024
Да. Но по сути дела тебе не надо смотреть. Если ты как-то сравнишь часы с быстродвижущимся объектом, то обнаружишь, что (в твоей системе отсчета) его часы медленнее. Но если это же сравнение провести в его системе отсчета, то результат будет ровно обратный.
408 502028
>>502027
А как сравнить? С помощью квантовой запутанности можно?
409 502030
>>502023
Потому надо генномодифицироватся. И нихуя не будет.
410 502031
>>502030
Ты так говоришь, как будто это очень легко.
411 502033
>>502023
Не следует забывать, что вся наша промышленность заточена на гравитацию
412 502034
>>502017

А относительно чего в ото и сто считается скорость? Там же смысл в том что универсальной точки отсчета нет.

Может ли быть тело у которого время почти не течет, типа 0.0000000001% от обычного времени вселенной без скорости и гравитации? Или я хуйню говорю?

У какого тела время идет быстрее всего во вселенной? какая у него скорость?

Или все настолько относительно что нет никакого эталона? А как тогда время устроено в целом во вселенной целиком? Или есть только локальные проявления уникального локального времени/гравитации/скорости?

Если вселенная раньше перед бв была сжата до размера атома - какая у него была гравитация, плотность, время? Ладно, знаю, никто не знает, это сингулярность.
Но через 1сек после бв - плотность вселенной была больше чем сейчас и общая гравитация и пространство совсем другие, соответственно и время текло по другому чем сейчас мы считаем? И возможно ли что те 1 сек после большого взрыва время шло со скоростью намного медленнее чем у нас сейчас и та 1сек это как 100млрд лет для нас сейчас?

Помоему я побил рекорд тупых вопросов
413 502053
>>502023

> Если мы будем жить в невесомости - мы превратимся в уродцев.


Заебись же.
414 502060
>>502053
Под уродцем я имел в виду нежизнеспособное существо.
415 502061
>>502034
В первое время своего существования вся Вселенная была размером меньше атома. Ты можешь себе такое представить?
416 502063
>>501975
То вода сколлапсирует в чд
417 502064
>>501960
А есть ли аналог зоны златовласки для каменистых пород? Ну то есть зона, в которой не вода находится в жидком состоянии, а камень.
418 502065
>>502027
Чому так? Нихуя понять не могу
419 502066
>>501789
>>501768
Мантию расплавим.
420 502071
>>501768

>ее один хуй сдует солнечный ветер у тебя же не магнитного поля


магнитное поле переоценено
421 502113
>>502064
Что мешает ей быть?
422 502176
>>502031
Легче чем строить орбитальный лифт. Как минимум при данных скоростях развития космонавтики\материаловедения и биологии. Особенно если строить этот лифт на каждом объекте.
423 502183
>>502176
Старые пердуны не допустят экспериментов над людьми. Вон, того китайца, который ВИЧ-резистентных близняшек сделал, китайскому правительству чуть ли не под охрану, замаскированную под арест, пришлось брать.
424 502203
>>502183
Просто сейчас ничего неизвестно толком. Когда всё будет известно какой ген, какие признаки делает и с каким геном кооперирует, никакие леваки не запретят. Как минимум учёные модифицируют сами себя. А истерия по поводу ГМО думается закончится, не в этом так в следующем подрастающим поколении.
425 502211
>>502113
Не знаю. Вдруг это слишком близко к звезде в паре километров там, например, где уже предел роша
426 502222
Бамп тупым вопросам по теории относительности

Относительно чего в ото и сто считается скорость? Там же смысл в том что универсальной точки отсчета нет.

Может ли быть тело у которого время почти не течет, типа 0.0000000001% от обычного времени вселенной без скорости и гравитации? Или я хуйню говорю?

У какого тела время идет быстрее всего во вселенной? какая у него скорость?

Или все настолько относительно что нет никакого эталона? А как тогда время устроено в целом во вселенной целиком? Или есть только локальные проявления уникального локального времени/гравитации/скорости?

Если вселенная раньше перед бв была сжата до размера атома - какая у него была гравитация, плотность, время? Ладно, знаю, никто не знает, это сингулярность.
Но через 1сек после бв - плотность вселенной была больше чем сейчас и общая гравитация и пространство совсем другие, соответственно и время текло по другому чем сейчас мы считаем? И возможно ли что те 1 сек после большого взрыва время шло со скоростью намного медленнее чем у нас сейчас и та 1сек это как 100млрд лет для нас сейчас?

Помоему я побил рекорд тупых вопросов
427 502223
>>502222

>относительности


лол суть относительности как раз в том что все считается относительно наблюдателя, неважно реального или абстрактного.
428 502250
>>502065

>Чому


Так получилось. Теория (в данном случае СТО) не объясняет почему, она предлагает модель того, как все работает. Модель отлично согласуется с результатами экспериментов и прочими наблюдениями. Правда, СТО не работает в микромире (тут нужен квантмех) и при сильных гравитационных полях (тут ОТО рулит).
429 502253
>>502222

>Относительно чего в ото и сто считается скорость?


Относительно системы отсчета, связанной с наблюдателем.

>Там же смысл в том что универсальной точки отсчета нет.


Нет. И универсальной скорости тоже нет.

>Может ли быть тело у которого время почти не течет, типа 0.0000000001% от обычного времени вселенной без скорости и гравитации? Или я хуйню говорю?


У любого тела со своей точки зрения время нормально течет (если не считать эффекты охуенной гравитации, это уже к ОТО). Время может замедлиться только с точки зрения кого-то. Если ты будешь в звездолете, летящем относительно земли на скорости охулиард км/с, то с точки зрения землян ты станешь слоупоком. А с твоей собственной точки зрения ничего не изменится - как кончал за две минуты, так и продолжишь.
430 502264
>>502253
Спасибо, многое прояснилось.
431 502279
Ананасики, а кто-нибудь знает, какие азимуты запусков разрешены с Восточного? Или там ваще нет таких строго определённых азимутов, как для Байконура?
432 502305
>>502279
ехал лахта через лахта
видит лахта в лахта лахта
сунул в лахта лахта лахта
лахта лахта лахта лахта
433 502371
>>502279
Не получится нормально запустить ракету на наклонение, меньшее чем широта точки запуска. Для Восточного это 51.5 градуса, т.е. уже на 40 градусов например хуй запустишь.
434 502384
>>497985 (OP)
Сколько звезд в видимой части Вселенной?
435 502386
>>501960

>гигантскую каплю воды


>радиусом в 1 км


Это не гигантская. Такая просто вскипит и испарится, при околонулевом давлении.

Чтобы такая капля была более-менее стабильна и была бы пригодна для жизни, она должна быть хотя бы размером с Марс, чтобы поддерживать атмосферу (из водяного пара, например).
436 502391
>>502371

>уже на 40 градусов например хуй запустишь.


Почему?
437 502392
>>502384
много, очень много
438 502401
>>502391
Придется менять наклонение уже после вывода на орбиту, а это дорого пиздец.

Не то что бы хуй запустишь, конечно, тот анон слишком сгустил краски, но изменение наклонения на 11 градусов это ~1500 м/с дельты, что весьма немало. Фактически целый разгонный блок нужен.
439 502416
>>502384
Сто миллиардов миллионов.
440 502430
>>502416
На 5-6 порядков занизил.
441 502449
>>502384
Одно только количество галактик лежит в пределах 10^12 - 10^13. И в каждой из них в среднем 10^11 - 10^12 звезд. Итого, количество звезд получается от 10^23 до 10^25. И это только по прикидкам. Может быть еще больше.
442 502460
Есть ли какие-нибудь процессы во Вселенной кроме коллапса звёзд, которые могут производить чёрные дыры? Ну там какие-нибудь столкновения частиц в джетах квазаров или во время гамма всплесков.
443 502477
>>502460
Есть теории, что ранние СМЧД образовались сразу, без звездной стадии, из газовых облаков. Потому что те СМЧД, которые мы наблюдаем в самых далеких, а значит и самых молодых галактиках, просто не успели бы образоваться "классическим" способом.
444 502478
>>502477
А так еще слияния нейтронных звезд.
33095725d73a07c.jpg46 Кб, 450x320
445 502496
Почему еще никто не сделал на орбите катапульту? раскручивается электромотором от солнечной энергии и пуляет грузы куда угодно. Топливо нужно только чтоб поддерживать высоту орбиты. В чем проблема?
446 502505
>>502496
Третьего закона Ньютона и закона сохранения импульса не существует.
447 502508
>>502505
Ты про то что катапульта будет улетать? Ну так крутить ее в другую сторону. Один аппарат запустили по часовой стрелке, другой против.
15657218384940.gif102 Кб, 300x300
448 502526
почему существует красное смещение? как оно работает?
земля, голубая планета, с марса выглядит зелёной звёздочкой
солнце, имеющее белый цвет, в полдень выглядит белёсым, на закат красным. почему? и почему солнце на рассвете красным не выглядит красным?
449 502531
>>502508
Не могу сказать, где в таком плане ошибка, но где-то точно присутствует попытка наебать физику, даже если в разные стороны швырять.
450 502532
>>502526
Это не красное смещение, а оптические эффекты в атмосфере. А красное смещение возникает, когда объект от тебя отдаляется. Грубо говоря, это работает так, что энергия света распределяется на большее расстояние, соответсвенно, понижается энергия волны, то есть увеличивается её длина.
451 502533
>>502532
Бля, не так маленько Объект не просто отдаляется, а само пространство между вами растягивается. Дальше все по прежнему.
452 502543
>>502505
А если гироскопами ее стабилизировать?
453 502545
>>502060
Я тоже.
454 502548
>>502533
Когда объект перемещается в пространстве и испускает волны, их длина будет слегка увеличиваться позади объекта - к ней будет прибавляться расстояние, пройденное объектом за 1 период колебания испускаемого им излучения.
Волна, испускаемая впереди объекта, будет наоборот, укорачиваться.
Какие причины этого эффекта - движение объекта или расширение пространства - неважно.
455 502550
>>502508
Вот короче смотри. Первая космическая для земли 7.91. Вторая 11.2.
Тебе надо запустить 2 болванки со скоростью 7.91 и за счёт второй ускорить первую на 3.29.(км\с соответственно). Звучит не очень выгодно. Я не считал, но звучит контринтуитивно.
Но даже если и да, то не забывай вот о чём: 1. Энергоэфективность солнечных панелей. 2. энергоэфективность этой вашей теслы. 3. Невероятно сложные конструкторские трудности, ради незначительного профита.

Сначала докажи что это эффективнее химических ракет, при условии условно бесплатной солнечной энергии и нулевом отклонении. Посчитай выгодно ли доставлять 2 болванки. Область применения и т.д.
Потом учти все нюансы вроде топлива для равновесия и примерной массы.
И уже тогда мы подумаем принимать ли твой проект в Омское КБ или нет.
456 502554
Какого размера должен быть круглый разворачиваемый на орбите Земли парус, чтобы он создавал постоянный солнечные затмения чтобы было видно корону?
Возможно ли такое сделать?
457 502561
>>502550
А еще надо прикинуть, какие перегрузки будет получать запускаемая хуйня, чтобы ускориться на 3,3 км/с. Причем космические ебалы делают не из цельнолитого чугуния, ведь массу надо экономить.
458 502566
>>502508
Ты наркоман штоле, сука? Если ты запустишь в противоположную сторону, то аппарат замедлится и упадет на землю. А ведь тебе нужно было топливо чтобы вывести его на орбиту. Короче хуита это.
459 502567
>>502545
А что хорошего в нежизнеспособности?
460 502570
>>502567
Хорошо, что нормальные люди не думают, что жизнь без гравитации означает жизнь в невесомости, а жизнь в невесомости обозначает смерть для всей жизни, а не только той что образовалась при земных условиях.
>>502566
Он имел ввиду одну полезную нагрузку, и вторую болванку которую можно и на землю скинуть. Один объект отлетит от второго и приобретёт скорость выше второй. Импульс передастся второму объекту и его компенсацией занимается третий объект который улетает в обратную от направления орбиты сторону. В итоге 1 улетает, 2 остаётся на месте, 3 падает на землю.
461 502571
>>502570
И как ты не в невесомости собираешься жить? Только не надо мне говорить про крутящиеся полые бублики, там сила кориолиса все будет руинить.
462 502572
>>502570

>и вторую болванку которую можно и на землю скинуть.


Её надо предварительно запустить в космос. Т.е. передать ему этот же импульс. Ты никак не обманешь закон сохранения импульса и энергии.
463 502576
>>502571
Большой бублик сделать
464 502577
Можно ли в теории сделать легкую очень прочную межпланетную станцию-кубсат и для экономии топлива и денег на разгон до внешней солнечной системы запустить ее сильным взрывом со 2 ступени, чтобы они разлетелись в разные стороны и легкая станция приобрела огромную скорость и быстро долетела до Юпитера? То есть по сути космический пистолет-пушка, запускающий станцию-заряд с начинкой во внешнюю солнечную систему. По идее станция должна выдержать резкий сильный разгон, тк боевые заряды с умной начинкой выдерживают же такой же разгон в орудиях на земле.
465 502578
Бамп

Возможно ли такое сделать?

Круглый разворачиваемый на орбите Земли парус несколько сотен метров, километров, чтобы он создавал постоянные солнечные затмения, чтобы было видно корону?

Какого он должен быть размера?
466 502585
А могут ли существовать черные звезды ? То есть настолько огромные что скорость убегания выше света, но не являющиеся чд
467 502589
Сейчас по авторитетному телеканалу в программе где борются со всякого рода мифами, лженаукой и переписыванием историей сказали, что наша вселенная на самом деле огромный мяч и ученые об этом давно знают, просто врут. И даже есть фотографии с телескопов. Размер вселенной примерно 40 миллиардов световых лет и она представляет собой додэкаэдр с 12 гранями, так вот каждый из этих граней просто напросто отражает свет в соседние и поэтому создается впечатление, будто вселенная бесконечна. Так показали еще очень известного азиата с длинными волосами, я его в этих тредах видел, уж он бы врать не стал!
image.png870 Кб, 670x670
468 502591
>>502589
Такой бред бывает когда рабочие теории с кучей неюансов пытаются довести до необразованного зрителя
Так понимаю они хотели рассказать о наблюдаемой вселенной, так вот, границы ее это те места, где объекты удаляются в следствии естественного расширения пространства от нас на скорости выше световой, поэтому мы физически не может увидеть что там происходит

>представляет собой додэкаэдр с 12 гранями, так вот каждый из этих граней просто напросто отражает свет в соседние и поэтому создается впечатление, будто вселенная бесконечна


это бред ебаный

>известного азиата с длинными волосами


вырвано из контекста наверняка
>>502585
нет, так как такая масса неизбежно сколлапсирует в дыру или прочее
есть черные карлики, по сути это просто остывшее ядро
469 502593
>>502591
кто тебе платит за ложь?
470 502595
>>502593
бородатый с пбс спейс тайм
471 502612
>>502577
Еще один (вместе с >>502496) желающий обойти законы сохранения? Откуда энергию на взрыв брать будешь? Из топлива? Тогда точнее и проще делать серию маленьких взрывов, переходящих один в другой и сливающихся в непрерывное горе... Ой, я только что ЖРД.
472 502629
>>502612
Так может масса взрывчатки или мини атомной бомбы для взрыва может эффективнее превращаться в кинетическую энергию межпланетной станции, чем ракетное топливо? В этом смвсл идеи.
1т взрывчатки выигрывает по конечной скорости и энергии, чем 1т ракетного топлива. в моей манятеории
473 502631
>>502554
Расстояние до Солнца - 150 млн км. До космоса - 100 км, то есть в 1,5 млн раз ближе. Значит, диаметр паруса в 1,5 млн раз меньше диаметра Солнца (1,4 млн км) - почти километр. Правда, на высоте 100 км еще слишком много атмосферы, тем более что у паруса большая, гм, парусность. Если взять 200 км, то диаметр паруса будет 1900-2000 м.

Правда, не забывай, что он должен вращаться, ну или затмевать Солнце только в двух точках. В любом случае, он должен быть дохуя прочный при таком-то размере (какие-то следы атмосферы есть и на 200 км, и даже на 1000 км) и каким-то образом не терять форму.
474 502634
>>502629
https://en.wikipedia.org/wiki/Energy_density
нет, взрывчатка очень сильно сосет у любых других веществ. единственное ее достоинство это скорость высвобождения энергии, и это нахуй не нужно во всех случаях когда нужно что-то взорвать
475 502637
>>502629

>Взрывы ядерных минибомб


Поздравляю, ты изобрел "Орион"
476 502643
>>502631
Еще один момент. Солнце под любым углом имеет один и тот же видимый размер, потому что изменением расстояния в пару тысяч километров смело можно пренебречь. А вот для твоего паруса будет совершенно не так. Максимальный видимый размер будет достигаться в зените, а потом быстро уменьшаться по мере того, как парус быстро съебывает. Еще и форма с круга будет меняться на сплюснутый эллипсоид. Короче, надо выводить на очень высокую орбиту, чтобы этого избежать, но тогда необходимый размер станет совсем уж конским - например, для геостационарной орбиты диаметр будет около 350 км.
477 502644
анон, как ты думаешь, возможно ли использовать солнце как семафор? заслонять его частично чем-нибудь и это далеко увидя, если смотрят сюда.
478 502649
>>502644

>возможно ли использовать солнце как семафор?


>1) Семафор железнодорожный — мачта с подвижными крыльями; сигналом является определенное взаимное расположение крыльев


Не думаю.
479 502651
>>502631

>Или затмевать солнце в двух точках



Вот этот момент не понял.

Ну то есть в целом это возможно?
Разворачивать парус вращением и центробежной силой, поддерживать вращение и орбиту двигателями.
Теоретически такой проект возможен? И наверно не слишком супер дорогой.
480 502652
>>502576
Большой - это какой радиус?
481 502656
>>502652
Если не путаю метров сто радиус норм будет
482 502659
>>502656
И из чего ты сделаешь стометровую крутящуюся йобу? По проблемам с прочностью материалов это тоже близко к космическому лифту.
483 502661
>>502651

>>Или затмевать солнце в двух точках


>Вот этот момент не понял.


Если твой парус не вращается, то он периодически (два раза за виток орбиты) будет становиться к Земле ребром и нихуя затмевать не будет. И полностью обращен к Земле будет только в двух точках, в остальных - под углом.

>Ну то есть в целом это возможно?


Ну да. Теоретически.

>Разворачивать парус вращением и центробежной силой,


Какой силой нахуй? Такой силы нет, погугли. Если ты имеешь в виду, что за счет вращения (один оборот вокруг оси за оборот по орбите) он будет поддерживать форму, то пожалуй да. Но только в одном направлении - поперек орбиты (и вдоль оси вращения) его ничего удерживать от скукоживания не будет, если он не жесткий.

>вращение и орбиту двигателями.


Ни то, ни другое двигателей не требует. Правда только если пренебречь атмосферой, так-то даже непарусную МКС периодически приходится подталкивать двигателями.

>Теоретически такой проект возможен?


Да.

>И наверно не слишком супер дорогой.


Вообще-то, звучит охуенно дорого. И нахуй ненужно. Еще см. >>502643
484 502663
>>502661
У МКС огромная парусность из-за солнечных панелей площадью более 2500 м², их на ночь даже поворачивают горизонтально, чтобы сопротивление воздуха уменьшить.
485 502666
>>497991
Будешь перекатывать, как хотел?
486 502671
>>502667
Телепатов нет, покажи на картинке, что ты имеешь в виду.
487 502681
>>502667
Темная материя.
488 502682
>>502667
Это пыль.
489 502693
>>502567
Вымрут всякие узколобые дурачки.
490 502740
>>502663
Ну тем более тогда. У такого паруса будет еще в 300 раз больше, правда он должен бы лететь "вперед ребром".
image.png998 Кб, 900x900
491 502744
Что спейсач думает про проснувшуюся СМЧД в центре Млечного пути? Что стало причиной и чем может нам грозить?
492 502745
>>502744
С хуя ли ты решил, что она проснулась, и уж тем более спала?
493 502747
>>502745
Ну так светимость резко выросла. Согласен, проснулось тут несколько некорректно, видимо засосала что-то крупное. Но что? Орбиты звезд летающих вокруг нее известны, ничего не предвещало.
494 502750
>>502745
Спящими называют ядра галактик, которые не являются квазарами.
495 502751
Гравитация это же свойство пространства типа ? Почему тогда пытаются искать гравитоны если по идее для искривления пространства никакие передатчики взаимодействия не нужны
496 502752
>>502751

>Гравитация это же свойство пространства типа


Согласно ОТО. Это теория, а не факт. Дает охуенно точные результаты, но есть и некоторые проблемы. Как оно там на самом деле - хз.

>Почему тогда пытаются искать гравитоны


Потому что пытаются построить "теорию всего", объединяющую и корректирующую ОТО и квантмех.
497 502757
>>502747

>Но что?


От тех же звезд потихоньку вещество приливными силами перетягивает, или газо-пылевые облака, или еще какая не особо заметная ебола.
498 502791
>>502589
в основном модно считать что Метагалактика в форме тора
499 502802
>>502752

>Это теория, а не факт


Теория, чмо, это высшая форма научного знания
500 502834
Недавно произошел ХЛОПОК и Стрелец А* сожрала какую-то хуету и начала срать радиацией. Вроде говорили, что джет направлен в сторону нашей плонетки, это пизда? Разве другие объекты на заблочат ненавистную радиацию?
501 502847
>>502834
Никакого джета не будет это не квазар
502 502864
>>502847

>не будет


Твердо и четко? энивей, джет был бы перпендикулярен плоскости галактики
15623998079810.jpg79 Кб, 853x1280
503 502872
Посоветуйте научпоп "Обо всём ", охватывающий основные понятия космоса и отдельно этапы освоения космоса(т.е истоки, программы и тд) и отдельно о советском\российском космосе.

спасибо
504 502878
>>502872
Обо всём - значит ни о чём. Читай бэ
505 502880
>>502847
Стадию квазара наша СМЧД прошла миллиарды лет назад.
506 502882
>>502864

>энивей, джет был бы перпендикулярен плоскости галактики


Эцхуяли.
507 502884
>>502872
Ютуб - Владимир Сурдин, Попов (реальные увеные и популяризаторы)

Программа Космос: Пространство и время с Нилом Деграсом Тайсоном.

Этого хватит чтобы покрыть 90% знаний в астрономии для любителя и понимать почти все процессы (которые известны) до которых может докопаться любитель.
508 502904
>>502882
Тому що СМЧД - плоть от плоти вещества галактики, и иметь сильно другой вектор вращения не может. 4млн солнечных масс более чем достаточно, чтобы все локальные отклонения обнулились, и результирующий момент импульса векторно совпадал с галактическим.
509 502944
>>502904
Большая часть этой массы наверняка старая, за миллиарды лет могло уже и повернуться, как солнечная система, например. Хотя хз, может и не могло.
510 502946
>>502944
Хотя как там прецессия оси вращения вообще работает? Наверное совсем с нихуя рандомный вектор действительно возникнуть не может.
511 503032
>>497985 (OP)
"Хаббл" как космический телескоп неплохо так выиграл за счет возможности пару раз слетать на его обслуживание. Что мешает воткнуть телескоп на МКС, дабы еще больше упростить обслуживание, но все равно иметь какие-то преимущества космического телескопа? Знаю, что размещение телескопов на космических станциях было, но вроде не сильно распространилось позже.
512 503037
>>503032
Заметно хуже качество будет.

На МКС 24/7 работает разная срань, создающая вибрации, двигаются люди, плюс внешняя обшивка засирается двигателями прибывающих и отчаливающих кораблей — Куполу даже закрывают каждый раз, чтоб иллюминаторы не зашкварить.

Космическому телескопу все это ни к чему, движущихся частей там должно быть по минимуму, а людей не должно быть вовсе.
14972916278012.jpg691 Кб, 1920x1280
513 503062
так бля поцаны этот вопрос наверн уже задавали но как вам идея строительства космической колоний? она будет дешевле колоний марса? ну имеется ввиду марс же так дохуя далеко а вот спейс колони вот тут прям за луной в точке лагранжа!

и еще вопрос я почему то думаю что в космосе строить легче. Правильно думаю?
15664970204270.jpg52 Кб, 500x764
514 503077
>>503062
Хули у этого корабля радиаторы так раскрываются, они же будут обратно в корпус тепло излучать. Или это не радиаторы?
515 503084
>>503037

>На МКС 24/7 работает разная срань, создающая вибрации, двигаются люди, плюс внешняя обшивка засирается двигателями прибывающих и отчаливающих кораблей — Куполу даже закрывают каждый раз, чтоб иллюминаторы не зашкварить.


>


>Космическому телескопу все это ни к чему, движущихся частей там должно быть по минимуму, а людей не должно быть вовсе.



Понятно, что это будет хуже, чем отдельный космический телескоп. Но это все-таки будет космический телескоп - без атмосферной турбулентности и поглощения спектра. Не будет ли куда более доступное обслуживание точнее - возможность обслуживания вообще компенсировать недостатки, вызываемые наличием людей рядом? Все описанные негативные условия в любом случаее слабее, чем на земле.
516 503092
посмотрел это https://www.youtube.com/watch?v=zOEcdclO-Bc и задумался - можно ли добить столкновения двух планет так, чтобы они не разрушили друг друга ни целиком, ни поверхности друг друге разъебали, а просто лежали, так сказать, друг на друге?
517 503093
>>503092
Я гуманитариеблядь и может быть не прав, но сила гравитации увеличится и сожмет всё равно в одну планету. Так что нет, не может быть.
518 503096
>>503077
Может это они так полураскрыты. Посмотри на профиль справа.
519 503101
>>503092
Ну во первых все планеты вращаются и движутся с большой скоростью (а не как в этой игре), то есть спокойно лечь при столкновении они изначально не смогут, да и гравитация со временем сожмет их в одну сферу
520 503108
>>503037

>людей не должно быть вовсе


Двачую вот этого.
521 503109
>>503092
И шарообразной формы они не будут. При сближении планеты будут вытягиваться по направлению друг к другу из-за приливных сил. Короче, если радиусы планет R, то соприкоснутся они задолго до сближения на 2R.

Заодно, такие ебические приливные деформации расплавят все к хуям, даже если планеты движутся так, чтобы разлететься после сближения.
522 503138
Могут ли в космосе образовываться снежинки?
523 503152
524 503182
>>502591

>в следствии естественного расширения пространства


Не доказано нихуя, а у вас это естественно
Ученые на хую верченые и верить им без доказательств бред.
Бога нет, но есть https://ru.wikipedia.org/wiki/Больцмановский_мозг Смех смехом..
15289912066410.jpg14 Кб, 319x331
525 503230
>>502878

>Обо всём - значит ни о чём. Читай бэ



ересь уровня квазиманятеоретиков орущих что для общего понимания квд нужно выдрочить тома квантовой механики, и в обще нечаво быдлу объеснять.

У меня была советская энциклопедия на страниц 600 охватывавшая от а до я космос и его освоение, лучшего базиса было не найти, а ты предлагаешь человеку интересующемуся космосом на уровне любителя - взять одну конкретную тему о которой он не имеет представления и дрочить только её, отбрасывая космос в целом

мдауж
526 503324
>>503092
Это сработает только с мелкими объектами, типа ultima tule
Тред утонул или удален.
Это копия, сохраненная 6 ноября 2019 года.

Скачать тред: только с превью, с превью и прикрепленными файлами.
Второй вариант может долго скачиваться. Файлы будут только в живых или недавно утонувших тредах. Подробнее

Если вам полезен архив М.Двача, пожертвуйте на оплату сервера.
« /spc/В начало тредаВеб-версияНастройки
/a//b//mu//s//vg/Все доски